You are on page 1of 117

MECH3410: exam answers

Why is all the text purple now? I hate whoever did this

Here is a different doc with more people in it (includes 2017 and 2018 exams):
https://docs.google.com/document/d/1eGvCWfNlOYanL3qJoirMHlRN-
naYObHjutnBXmSZ4bQ/edit#

2008-2016 Exams zip: https://drive.google.com/file/d/1lKHWjKieb02_8YOiBO6eJ8IZ1OCEYhp5/view?


usp=sharing

2017
Part A

Part B

2016
Mess Lecture on Blackboard
Part A
Q1: (e) +1

Q2: (b) B[6] Answer is 2.132356014 N mm.


-Account for relative velocity to wind
-Because cup facing into the wind is 17.5 ms^-1; Cd = 1.4 (3D cup shape)
-Back facing cup is 22.5 ms; Cd = 0.4 (3D cup Shape, reversed)
-Torque arm length = 50mm.
-Forces are acting in opposite direction, therefore subtract them.
C [2]
Q3: (a) [5] In the Mess session the tutor used water, but the question says air. (e)
I get 0.443 which is close to (a) is it safe to assume (a) is the answer? I’m getting 0.443 also
(agree: 1) na boys…. Ya all wrong….. Answer is 0.4488957757 rekt I’m getting 0.42m/s, are
y’all using the flat plate approximation?. Someone hav worked solutions pls

Q4: (a) Agree: 8


(f) - I think the tutor’s rounding messed him up, I get 392 Pa with f=0.0263 I got
f=0.02667328779 which gives me (a) (Agree: 8)
I agree with A, but got f = 0.02960 giving deltaP as 443.112 Pa
Yeah I redid it just now (1 week later) and got A. Dunno what I did back then. Maybe my own
rounding messed me up? haha

Q5: a) False → because axial turbines are better suited for high flow rates
b) False
● Once the stall angle is exceeded, stall occurs → causes massive decrease in lift due to
flow separation on the top surface of the wing.
c) True
● The flow may either speed up or slow down depending on if M<1 or M>1, and
properties might change with it.
d) False (+4)
● Area change is isentropic → no stagnation pressure drop
● Friction (Fanno flow) always causes stagnation pressure to drop
● Cooling (heat removal) always causes stagnation pressure to increase
e) False (+4)
● Friction (Fanno flow) always causes stagnation pressure to drop
● Shocks are not isentropic → stagnation pressure always drops
● Cooling (heat removal) always causes stagnation pressure to increase

Q6: (d) +6 Use figure B.2, given that we know P1 and P2, you can use the pressure ratio
which is P2/P1 = 1.5, and we know the angle of beta(B) = 45. So from the graph, it
approximately around 1.7 for Ma1-
Another way... p2/p1 = 1.5 from tables this corresponds to Mn1 = 1.2, 1.2/sin(45) = 1.7

Q7: (d) M1 = 5, T1 = 300K, find M2 u

sing
This will give M2 = 0.415

Now work out T2 as well given we know what T1 is using the temperature ratio

This will result with T2 = 1740K, now we can find a2(sound speed) which is sqrt(1.4*R*T2) =
836m/s

V2 = M2*a2 = 346m/s
We have to take reference frame into account and the question says behind the shock wave
in laboratory reference frame.

This is Vrel = Vs - V2. where Vs = M1*a1 = 1706m/s


So Vrel = 1706 - 346 = 1390m/s

The above method is very long, easier way to do it:


Vs=Ma=5*sqrt(1.4*287*300)=1735.94 m/s

Vs/V2=V1/V2 -> using tables find V1/V2=5


Therefore: V2 = Vs/(V1/V2) = 347.188 m/s

To find the reference frame of Lab:

VL = Vs-V2=1388.751 m/s
Agree…. Above works well… fair easier
Q8: (b) w should be relative to rotor, u = omega*r which is rotor circumferential velocity, idk
how people do it but V should be the velocity in absolute frame, or the vector between w
and u

So something like this

So B has this triangle

Here's a pic if you need some help with velocity triangles

Part B
Question B1
Edit: for part d of B1, the tutor made a mistake of the Cd_infinity, he said that Cd_infinity is
0.075 but it should be 0.0075 actually
Answers given on Platypus

“C” and “D” answers different to what the tutor did?? tutor used momentum integral, above
uses Blassius. Also tutor read the chart wrong for Cd_inf, he used 0.075 instead of 0.0075.
Drag force on the wing seems to be the same, however drag force on the plates is different?
What length of L was used in the integral in the solution above? Do we need to use a length
other than b=0.04, as the tutor used? OR is the difference in not multiplying by H above?

Question B2
Question B3

Question B4
Vf

2015
` Part A
Q1: (Ans = C) Agree: 26
I get 2.01 but 1.9 is still closest I get 1.93 so dunno what youre doing wrong
Solution:
V = √❑
0.01= √❑

Q2: (Ans = B) Agree: 23


Solution:
Q=VA → V =Q / A=4 Q /π D2=12.73 m/ s
ρV Lc 960 × 12.73× 0.01
ℜ= = =122.208, Where Lc is the characteristic length, in this case D
μ 1
64
f = =0.5237

L V2
h f =f =86.511
D 2g
V 22
P= ρ g( + h )=892502 Pa=8.92 ¯≈ 8.09 ¯¿
2g f
Why is frictional loss considered when they said the pipe was smooth and minor losses are
ignored? Cos frictional losses aint minor losses m80.
A “smooth” pipe still has frictional losses. Check out the moody diagram. (Agree)

Q3: (Ans = E ) Agree: 25


Solution:
FD mg
CD= → A= =136.25m2
1 1
ρ U2 A ρ U2CD
2 2
2
A=π r =136.25→ r=6.6 . m
Where C D =1.2 from “Drag Coefficients (3-D)” table for parachute. Assume no drag from the
man. If you include drag from the man as well, r =4.66 m which is not one of the answers.

Average person: C D A=1.2 f t 2. To get r =4.66 m. They have added this to the C D of the
parachute, which is wrong.

Q4: (Ans = A) Agree: 18


Solution:
Q 0.025 m 3 / s
C Q= = =0.4
n D 3 62.83 rad / s ×0. 13
Is n always rad/s? I thought it was rev/s
Use C Q to find C H =2 and η=0.8 from graph
gH n2 D 2 C H
CH= → H = =8.05
n2 D 2 g
1 1
P= ρ gQH = × 998 ×9.81 ×0.025 ×8.05=2463 W ≈ 2460W
η 0.8

Q5: (Ans = C) Agree: 22


Solution:
u: vector is same direction as rotor spinning
w: tangential to the blade
Q6: (Ans = B and E) Agree: 13
Solution:
(Ans = B and E) Agree: 11 - Check out lecture on 31/08 at 20 mins.

Why not 1A??????x? (Agree: 24)


Why not C?? Doesn’t the flow become subsonic as per the lecture slide
above? Agree
So is the only incorrect answer D? I.e. answer is A, B, C, and E?

Q7: (Ans = C) Agree: 19


Solution:
V V
M= =
a √❑
Table B.1
P1
=0.0066 → P0,1 =151.5 kPa
P 0,1
Table B.2
P 0,2
=0.1388 → P0,2 =21kPa
P 0,1
Can someone explain this with a diagram maybe? I kind of get that the shock forms before
the nose of the ship, meaning LHS of shock is P01 (151k Pa) and RHS is P02 (21kPa), so the
guy in red seems right. Because the pressure against the nose is on the RHS of the normal
shock. But yeah not sure if this is the correct way at looking at it
Yeah that’s right. On the LHS it’s Mach 4 so the stag pressure is 151.5kPa. But after the
shock it’s only Mach 0.435 so the stag pressure obviously less (21.03kPa).

Q8: (Ans =E ) Agree:14


Solution:
Table B.1
T1
=0.2381 →T 0,1=4200 K
T 0,1
q
T 0,2=T 0,1 + =5196 K
Cp
Table B.4
T 0,1 ¿
¿ =0.5891 →T 0 =7129.5 K
T0
T 0,2
=0.7288 → M 2=2.4
T 0¿
M 2 ≠ 0.5 as the initial flow is supersonic, and that heat addition will reduce M towards 1.0, but
not below it. Instead, weak and strong shocks relate to deflections in the flow.
Part B
Question B1
Answer from platypus week 10

a) Figure 9.23: β ≈34 Deg Agree: 14 (easier to use Figure B.1).


Figure B.1: M 1 ≈ 1.7−1.8 Agree: 14
How do u achieve 34 deg? The only way that’s possible is if u take M0 to be M1 because
Figure 9.23 uses M1? Irrelevant, because it’s just representative of the flow at states 1 and
2. M0 is M1 and M1 is M2.
Shouldn’t this be beta = 44? Taking the value directly across and not following the line on
the chart Seeing how Fig.B1 and 9.23 should correlate to each other, this would mean that
we have to achieve around 34deg for the former too, meaning that the only way to do that is
by reading along the curve
Alright ty mate no worries man
1
γ −1 2 ❑ 2
b) Normal Shock: M 2=
1+

[ 2
2
γ M1 −
M1
γ −1
2
]
=0.6405, Assuming M 1=1.7 Agree:14

M_2 = 0.877 , M_2 = 0.5956 (from M_1 =approx 1.9)


For M_1=1.85, used normal shock relation and got M_2=0.61 +1
I found normal component of M1 and it was 1.9 so Mn2=0.5130=M2

A3
c) M 3=0.15 → =4.3927
A¿
A
M 2=0.6405 → 2¿ =1.1413
A
¿
A3/ A A3
¿= =3.85 Agree: 11
A2/ A A2
Has anyone got 2.1216 for Mach number 1.8? no.

d) Table B.3
M 3=0.15 → P 3 /P¿ =8.19945 → P ¿=12.1959 → f 3 L/ D≈ 40.72745
P4 /P¿ =3.0338 → f 4 L/ D ≈ 3.419← how did u get this? For P4/P* = 37/8.19945 = 4.512
f Δ L/D=f 3 L/ D−f 4 L/ D → f =D / Δ L[40.72745−3.9639]≈ 0.74
ΔL =0.5m
Agree: 20

¿
Shouldn’t P3 / P be 7.2866?
If you linearly interpolate, you should get 8.19945. But linearly interpolating gives a
error since the rate of increase is large for low M numbers, so using an equation should
give the best result. > results in M4=0.4, and f = 0.51 . . f3L*/D=27.9, f4L*/D=2.3 … ? <
^ I got the same when using the equation to find p3/p* (f = 0.51).
Why not use ideal gas law to determine T_3 and then find Vavg with M_3=0.15. Then
rearrange energy equation to find f?

fL/D p/p*

66.9216 10.9435

40.7275 p3/p*

14.5333 5.4554

e) Friction and heat both drive the flow to M=1. Less heat will be able to be added
compared to a frictionless combustor. Wouldn't that mean more heat can be added
since there is no contribution from friction? With both in play it will choke faster. It’s
asking how the friction will affect the maximum amount of heat that can be added
and since both the friction and heat contribute to the flow choking, less heat is
required i.e. maximum heat that can be added is reduced.
Question B2
a) i) Use energy balance equation and rearrange
2
L V 2 , assuming V =0 , P =P =P
H 1−H 2=(1+ K + f ) 1 1 2 atm
D 2g
Therefore C 1=1+ K and C 2=L/ D

ii) C1 = 3.42, C2 = 800, f=0.024


C1 = 1+ sum(K) = 3.42 (+6) (Why k for sharp exit is not considered here?) It is
C2 = L/D = 800 ⇒ f = 0.024⇒ v = 4.0064 m/s ⇒ Q = 0.0177 (+7) Legendary Yankine <3
How did you find f to be 0.024?
Because the relative pipe roughness is 0.002 and the lowest friction factor it can have according to moody diag is 0.024.
Rearrange the eqn you just worked out for V then pick an f-valueaAZX. Then just repeat the
iteration process from Wk3.
ρVd
ℜ= =5000 ×V
μ
Should use Re No. to check if turbulent therefore confirming if Moody diagram f estimate is
correct.

iii) Q=VA =0.25 V π D 2=0.0177 m3 / s Agree: 15

b) i) The Pump adds energy to the system, thus increases the head
ii) Q = 0.06m^3/s. (+10)
(H1-H2)=(C1+fC2)*V^2/2g-(100+(25.1*10^3)Q^2) ->sub V^2=Q^2/A^2 -> solve for Q what is
the value of h1 - h2 ? I used H1=20m and H2=1.5m
I can only get 0.06m^3/s if I change the pumps performance to 100-(25.1x10^3) Q^2. Anyone
else get this??? → Yeah
Performance curves are always negative parabolas,
see google images for ‘pump performance curve’ or look at the equation
in Q4 Platypus 12 (the Q2 is negative)

iii) Re=1e6, from Ha aland f=0.0236 =~ 0.0235, appropriate


Isn’t iii) asking why we can assume a constant friction factor for high velocities? In which
case I would say that you can because as v increases, and therefore Reynolds number
increases there is little impact on Haaland’s formula. <- Agreed
Yeah, have a red hot look at the moody diagram its flat af at these high re values.
^Here here^
Question B3
a) Table B.1
P1 T1
At Ma=0.8 → =0.6560 , =0.8865
P0,1 T 0,1
P0,1=15.24 kPa ,T 0,1=248.17 K Agree: 25

b) Table B.1
M 1=0.8 → A 1 / A ¿=1.0382→ A ¿ / A1=0.9632
M 2=1.5 → A 2 / A ¿=1.1762 Agree: 21

c) Table B.4
M 3=4 →T 0,3 /T 0¿=0.5891
M 2=1.5 →T 0,2 /T 0¿=0.9093❑
T 0,2=T 0,1=248.17 K
T 0,3 T 0¿
T 0,3= ¿ × ×T 0,2=160.78 K
T 0 T 0,2
q=C p (T 0,3 −T 0,2)=−87739 W =−87.74 kW /(kg /s)
Similar process to find P0,3as shown for temperature.
P0,3 =111.79kPa Agree: 20

T_0 is the stagnation temperature


T* is thermally (in this case) choked
T. T*_0 is thermally choked T_0

P 0,3
d) =¿
Pe
Use Table B.1 to find T e /T 0,3for M e =2.23 →T e ≈ 80.62
M =V /a → V e =M e × √ ❑ Agree: 20,

The pressure ratio of pe/po=0.08945 therefore isn’t the flow choked?

e) V 1=M 1 a=M 1 × √ ❑
F
=V e −V 1=163.5 m/s Agree: 15
m

f) Table B.1
M 3=4 →T 3 /T 0,3 =0.2381 →T 3=38.28 K <77 K Agree: 17
Question B4
FL
a) C L= =0.3997
0.5 ρU 2 A
C L ,∞ =0.4049
α =4 Agree: 20

As per the blackboard discussion thread: "Wing has a planform area of 22.5m^2"
refers to the entirety of the airplane wing. I.e. left half + right half

Aren’t those CL and CD graphs for infinite wings? Do we need to convert calculated
CL into CL_inf, and then find alpha from graph. CL(inf) = 0.4045 so same answer but
yes you do << is CL(inf) not 0.4? As it is dimensionless - so we find a finite CL =
0.395, obvs minor difference but yeah - aren’t the graphs are for finite wings not
infinite? Why would it be an “infinite wing” if AR is introduced here.?? Not all
questions have such close CL and CLinf so you have to convert between the two the
find the angle of attack

b) Fd = 189.3 N (+10)
Fd = 197.82 N
Fd = 270.02 N (<- The other answers seem to use b=15, whereas b,single_wing =
6.5m. This results in Cd,wing of 0.018733, not 0.01209. Has aspect ratio been defined
including the fuselage / both wings in lectures?)

From Aerofoil graphs:


Rechord = Rho U c / (dynamic viscosity) = 1.2 * 30 * 1.5 / (1.8*10^(-5)) = 3x10^(6) This is Rec
(Reynolds number based on wing chord - Refer to tutorial Week 10)

C D ,∞=0.007 From graph for ℜ=3 ×1 06, lowest point of that curve for is at C D ,∞=0.007
C L2
C D ,wing=C D ,∞ + =0.01209
π AR
From “Drag Coefficient (3-D)” Ellipsoid
C D ,fuselage =0.1 as L/ D=4 and assuming turbulent flow
02W14hat Cd values are people using to get Fd=189.3? Any input would
be great...

Is Cd from graph not the finite Cd? Dimensions are accounted with AR so surely its not
Cd(inf)? Thereby getting Cd(inf) = 0.002 = Fd/0.5ApU^2… Fd = 24.3N ???? am i alone?yes
go home

For ellipsoid (3-D Drag coefficients) with L/D =4 and turbulent flow, Cd_fus=0.1 Calculate
respective drag forces and sum using the frontal area of the ellipsoid and the planform area
of the wing. >> Fd = 189.3 N (+9)

What is the frontal area? (x1) … pi*D^2/4 = pi/4


What area do we use to get the drag on the wing? Planform Area (22.5)

c)
Can you guys please explain how you got your answers? I’m having a little trouble
here
It says that you cannot assume a drag coefficient so you have to perform an integral
where F=integral(tau_w dA) and tau_w is a function of Re, you need to find that the flow
over the wingtip is laminar because x_cr=0.25. Then applying the Blasius solution and
multiplying by 2 for 2 wing flaps the total increased drag force is:
❑ ❑ 0.25
F=2∗∫ τ dA=∫ 0.5 ρU C f dA= ∫ 0.5 ρ U 2 H ¿ ¿ ¿ ¿
2

❑ ❑ 0
0.25
F=2∗0.332 ρ HU 3/ 2 ν 1/ 2 ∫ x−1 /2 dx=0.2535 N
0

Could be errors in there but I think that’s the approach he wants. ^ Should be half that.
Shouldn’t it be double that ie 0.507N because 2 sides * 2 flaps? (agree: 3)
Why do u use blasius? Blasius solution is the exact solution, so unless question states use
momentum integral estimate, we don’t need to use non-exact.
How do you know to calculate it from 0 to 0.25, rather than 0 to 0.5? That way I get F =
0.03N. Because planes fly forwards not sideways. Its 0 to 0.25 as thats the direction the air
is flowing, L_R, so its the x-direction for the boundary layer formation. The b direction can
be considered constant, so we just multiply by b to account for it, we do no integrate over
b.

2017 comments: I get 1.3 N for 2 flaps taking into account both sides of each flap - as pink
above. Used the turbulent momentum integral since Rex = 5E6, which is right on the
transitional reynolds number. (agree +1) Yeah I wasnt sure if we were meant to do Laminar
or Turbulent for this and its border line??You use laminar because xcritical is 0.25[+4]
meaning that it only becomes turbulent at the end thus its laminar across the whole thing,
when using laminar i get 0.4918N for total drag with all sides and rudders (answer*7)
I get 0.03169N for each side and therefore (x4) 0.12677N in total

d) 0.216, 174.944N (3), 207N (13) How are you lads getting 207N?
Using mean velocity to calculate the mass flow rate can anyone post a solution of this
here?

vBar = (v2+v1)/2
mdot = rho*A*vBar
F = mDot(v2-v1)how did you get this formula? When (kg/s * m/s = N)? Im confused.
??F = rho*A*(Vout-Vin)^2 = 64.147N it has the correct units ??
A newton is fundamentally kgm/s^2 - does this help? Thanks man.
Or you could think about it at momentum exchange F*delta_t = delta(m*v)

e) (there is actually a part e) i think the thrust increases velocity and therefore drag (+1)
however that seems like a cop out so anyone with a better idea is welcome.that and the
propeller itself would contribute to drag and the air coming from the propeller over the body
would be turbulent as fuck
-Reynolds number would increase in all cases as a result of the increase in velocity. Drag
coefficients and the Velocity^2 terms result in an overall drag increase.
2014
Part A
Q1: (Ans = C) Agree: 50

Someone plz tell me how to do this <3 +1 halp

Q2: (Ans = B) Agree: 22


How did you find that CL???
Look at the peak of the CL curve for the NACA0009 aerofoil for No Flap

Doesn’t make a lot of difference but should use this eqn to convert but to finite wing
(graphs use C_Linfinity)

Q3:(Ans = A) Agree: 36
Assumptions:
V 1=0 (tank ) , P1 =P 2=Patm , z 1=4 , z 2=0
Using bernoulli’s equation, solve for h f =3.352
ρV Lc
ℜ= =1491.13
μ
f =64 /ℜ=0.04292
L V2
hf= × → L=6.03 m
D 2g
D
Q4:(Ans = C (22.9kW)) Agree: 17
Power Extracted (Windturbines):
P=0.25 ρ A( V 12 −V 22)(V 1 +V 2)
P=0.25 ρ A( V 13❑ +V 12 V 2−V 1 V 22−V 23 )

Differential w.r.t. free stream velocity V 2 and equate to 0 to find max/min


dP
=0.25 ρ A(V 12−2V 1 V 2−3 V 22)
dV2
0=0.25 ρ A (V 12−2 V 1 V 2−3 V 22 )
0=V 12−2 V 1 V 2−3 V 22
0=¿
1
V 2= V 1
3

V =0.5(V 1 +V 2)
3 = ½(V1 + ⅓ V1) -> V1 = 4.5m/s
A=π R2 =0.25 π D2=706.86 m2❑

1
Substitute V 2= V 1 back into “Power Extracted” equation and solve
3
8
P= ρ A V 13=22.9 kW
27
F=rho*v^2*A ---> P=F*V = 22.9KW

Q5:(Ans = C) Agree: 6 not convinced as fuck


It’s b you retards, see lecture 4pm 14/9/16 section 1
Patching does not work when separation occurs, and there’s a drawing of a bullet
Im with red guy^. High Re may make patching better but patching wont work with
separation. Patching relies on boundary layers smoothly forming and transitioning (i think),
I think the correct answer is (ANS = B) as this is similar to the wedges demonstrated in
lectures.
Frank White FM Ed7: P457-460 - “Patching becomes more successful as the Reynolds
number becomes larger” "A high-Reynolds-number flow is much more amenable to boundary
layer patchin F) logical...

ANS = B [Agree: 7]

Q6 ??? B or C?We usually assume isentropic through area change (Po1=Po2) so id go with
B
ANS = B [Agree: 3]

Q7:(Ans = D) Agree: 34 yeah mofos


1) Use q = cp(T02 - T01) to find T02;
2) For M1 = 0.2, look at table B.4 T01/T0* = 0.1736 to find out value for T0*
3) Use the T0* and find the value for T02/T0*= ….
4) Look at table B.4 to find out M2 and you will get 0.3 (Therefore, answer is D.)

Q8:(Ans = C) Agree: 13 [Which lecture slide to refer?]


Part B
Question 9
a(i)
12.23 m/s +2
12.03 m/s +4
12.29 m/s +1
12.53 m/s +2
13.21 m/s +9
How we did it: the friction head is 20% of available head at the turbine inlet. This is
0.2(Height + v2^2/2g). Sub this in for Hf, include Hm, no Hpump. V1=0, P1=P2. Use energy
equation and rearrange for the only unknown which is V2 = 12.03 m/s (+2). We’re pretty
confident in it. hf=0.2(Height + v2^2/2g) Can anyone explain why you would not include
pressure head in the total head available? It says that the inlet to the turbine is at
atmospheric conditions - a fluid at atmospheric pressure is at pressure equilibrium and has
no ‘pressure energy’ (use gauge pressure in the head equation)

Hf would be like 0.2*(10-v^2/2g -0.1v^2/2g) -> minor losses as well

Why is the head of the turbine not included above equatioN**?

I think 2 should be used on the total head instead of 10 for hf agreed and have a positive
velocity
Wouldnt the theoretical head at the turbine be 2 + v2^2/2g? I think it's a typo above his
workings he mentioned sum too.

everyone seems to be doing 10 for z instead of 2? 12 - 2 = 10

According to the image above though, we should just be using 2m. Z1 = 12, z2 = 2. When
calculating theoretical head of pump youll use Hpump only which is 2m.

Q9:a ii)
Using a v = 12.19m/s, I get a diameter of D = 1.245m when iterated, corresponding to a f =
0.01383. +5
By using f = 0.02, V=12.037 and using our energy (bernoulli) eq. from before I got D1 =
15.738m. .
i get D=1.5738 m +3
Then I subbed this result into Re and used this and pipe roughness (10^8 , 0.25) to get a
new f value (~0.054). shouldnt roughness be .00025/D… I get f = 0.0137 +10086
Now riense and repat by using new f to get new D2 = 4.25m. By finding Re again (~5x10^6) I
got the same f value so co v nverge to answer of D=4.25m (+2). Getting the same
Using v=13.21m/s I get D=7.85 +4 (7m for pipe diameter, seems too reasonable?) -
Using v = 13.21m/s, D= 8.03m, by iterating D = 822.07f +2

D=1.5738 m +3
D=4.25m (+2)
D=7.85 +4
D = 8.03m +1
D = 1.102m
9b)
Using Q=10m/s, D=1.2m. I got k=1.159 and so value is 64% open.
I did this by using the Q and D to find a new V=8.84m/s. Do energy equation but we now
know the V and need to find a new K for the valve. Then use graph to convert.
Using D=4.25 and Q=10 (V2=Q/A=0.705m/s)
I used Bernoulli with frictional and minor losses. We know everything except for one of the
minor loss coeffictients, so hm = [0.05 + k_ball]*V2^2 / 2g.
Rearrange for k_ball. I also checked darcy friction factor with new velocity->new Re and it
still came out pretty close to 0.054. From that I got k_ball = ~393.33 which is large but just
fits on the chart we’re given so opening = 20%(+2) Using D = 1.125m, get v = 10.06m/s, this
gives K_ballvalve=2.9E-1 hence opening is roughly 85% (agree +3) How doesv = 10.06m/s?
V = Q/A = 10/(pi*1.125^2) = 2.515m/s(Radius mate)
i got K_ball of about 0.59 which is roughly around 85%
k=5.88 , 56%open when D=1.574m

9c)
P=omega.T=rho.g.Q.Hp. Ball valve slows flow, drops Q, P and Q are directly proportional,
therefore power output drops.

9D)ii) What are matched conditions? Is it something to do with alpha or radial velocities?
Isn’t it just the flow rate and the RPM, and then you get the rest from the geometry?
What do people get for this? I get 21.46 degrees

So I think “matched conditions” is just like the assumption that everything adds up. So if
alpha is the angle the absolute inlet velocity makes with the normal then the only thing we
don’t know is beta1 and we can solve for it using our usual velocity triangles given alpha =
70degrees and that Vt1 = u1 + wt1. ← matched conditions? if this is true I got

I get beta1 = 62.67 degrees, find Vr from alpha & V. beta1 = arctan((Vt-U1)/Vr).+1 except Vr =
Q/A, then Vt = Vr*tan(alpha), giving beta=45deg +9
What V are you using to get Vr? 5.65m/s from the previous questions (pipe flow velocity)?

Stupid question but does matched conditions mean the inlet and outlet velocities are
matched (V1 = V2)? Or are the relative velocities of the inlet and outlet equal? I get 55.68
degrees.
i get 37° for beta using tan-1((Vt-u)/Vr). Starting with Vr = Q/A1. Fairly confident about this
answer. How do you get Vt ??

V_T1=0 as this is a radial in-flow turbine, so V_1 = V_R1 so B_1=60.18 degree


Not true at all ^. Check lecture notes and velocity triangles (Yup, it’s supposed to be
V_T2=0) B_1=~45 deg

Also not true^ V_r1 = 3.6


V_t1 = V_r1 tan(alpha) -> From Geometry
U_1 = omega * r_1

Then V_r1 * tan(beta) = (Vt_1 - U_1)


= 45 degrees

Get Vr2=3.6 U2=6.283 beta1=54.1 +1.


Question 10
Typed solutions completed by MESS tutorial session.

`
I believe for part C that beta2=36 degrees from Figure B1 <follow the curved beta lines, do
not look for the beta value horizontally across>
It is right in between the lines for 30 and 35, so about 32 should be correct.
also T03 = 638+300+459 = 1397 K not 1260,bv agree +2(your error was accumulated due to
rounding) Do not agree - state 3 is before heat has been added. The stagnation
teemperature is the same as in th reservoir because shocks don’t affect stagnation
temperature.
Question 11
I calculated the Reynolds using Re = rho*v*L/mu at 10 metres and found the displacement
thickness here, then did the same thing at 30. Assuming that there is a sudden expansion at
the start of the test section to equalise the velocity back to 25m/s at 10 metres, I got a
height increase of (Ans = 58.17mm ) Agree: 9
for a), i think we should calculate boundary layer thickness instead of displacement
thickness. please check external flow lecture slide part 1, page 30.
I think so too, ‘free-stream air velocity’ is the region outside the BLs (or between them in
internal flows). So what I did was sum the BL thickness with BL displacement thickness for
both positions and found the difference. I got 0.2624m. This is quite large because as you
continue down the pipe, flow becomes more and more restricted, BL increases. Dump
Defs displacement thickness (Agree: 3)
Pretty sure you use displacement thickness (Agree: 4). According to external flow lecture 2,
delta* is used to calculated inviscid core flow properties i.e. velocity, whereas bounda
OP got 10.783N. +4l
Do we *2 to get the drag force on both sides of the flag? I think so [agree: 2]
If considering both side 21.568N [+3]
Do we have to interpolate for the CD, too much effort
Why is cd not just 1.2 aye? For bii) It’s explained in the top paragraph ^. Basically
D/L = 0.0075 so after interpolating the 3D Cd cylinder you’re with Cd = 1.134
b. v) 10.7N (single side) (+3) - I got 19.95N single side
b) ii) Shouldn’t the area be: ((2*pi*r)/2)*H? Frontal Area, not surface area
Frontal area’s right. what is that 10*0.075??? height times diameter? i think this ans
is wrong. also when you interpolate the inverse numbers, the corresponding
number for 1/40 is not 0.98. im not 100% sure, but at least i dont think thats the
case. simply draw a graph, and model it yourself.
Why 1/40?
b iv) how do you derive the expression 4-x
for b (i), why does an object have reynolds number? i thought reynolds numbers are
only for fluids. helps plz
Reynolds nuimber is the numbre of fluids passing the object per unit time
Reckon its the reynolds number of the fluid flowing past the object mate
Question 12
a) L=0.9816m Agree: 4
Table B.3
L 0.02 m
At M 1=0.6 → f =0.4908→ L=0.4908 × =0.9816 m
D 0.01

b) 6.16 kg/s Agree: 4


7.33 kg/s Agree: 5
6.22 kg/s Agree: 1
Plat-her-puss says 6.2kg/s
7.33 kg/s by using the choked mass flow rate equation from equation sheet

Okay you get 7.33kg/s if you use D=20mm and find the area, then put that area into
the formula on the equation sheet, but the choked mass flow rate specifies A* not A,
so you go to table B.1 find A/A* for M1=0.6 and then find A*, put it into the formula
sheet equation and get m_dot = 6.17kg/s

6.22kg/s by rho.v.A at the entrance of the pipe, ideal gas equation for rho, mach
equation for V, T1 and P1 from isentropic relationships to stagnation pressure and
temperature. Lecture 7/9 45min

Can confirm 6.17kg/s by using either choked flow equation or ideal gas equation,
make sure you convert p and p0 and such
f
or q12 b & d, i understand ur process to get the mass flow rate but why can’t we just
use the choked mass flow rate equation, as we assume the flow is adiabatic and
isentropic at the entrance?? anyone understand? orange is right, we get the same
answer as the above soln. orange is not right, because isentropic only applied from
tank to entrance, not after, i.e. in the pipe. But
I used rho*v*A right at the entrance which is 104.05kg/m^3 * 201.8m/s * 3 .14E-4 =
6.59
thanks !
So do we also use p1 or v1 instead of p* v* in our equations for q12b?

d) 2.1kg/s not 7.33kg/s (checked w


e) now config 1 is better
Does it make sense that the supersonic configuration is worse due to the
nozzle throat becoming the limiting area? (since the throat also has mach 1 flow) So
the density and velocity are the same (evaluated for the sonic condition) but the
areas are different.
2013
Part A
Q1 (Ans = D)
V 1L 5
ℜ= =250 000< R Ecrit =5× 10 → Laminar Flow
v
¿
δ 1.721 ¿
= → δ =0.01721m
x R e 0.5
H ¿ =H−2 δ ¿=0.16558m
Conservation of momentum
V 1 Hd
V 1 A 1=V 2 A 2 → V 2= =0.91m/s
H¿d
d remains the same, therefore cancels as flow is 2D

Can someone please explain difference between delta and delta*? Thanks. - THIS PLEASE!!!
I think delta is the boundary layer thickness and delta* is the displacement thickness, which
is the outward deflection of outer streamlines needed to satisfy conservation of mass
between the inlet and outlet. So delta* is displacement thickness of the boundary layer.
(easy to remember when you think * denotes critical values)

So what's the difference between boundary layer thickness and displacement thickness…
What the red guy above says explains it. It tells you the amount by which the area will
change so that you can satisfy conservation of momentum. Just memorise how it is in this
problem and in the tute example and that's enough.

when you calculate the cross sectional area, we dont even know the shape of the thing, do
we assume it is a rectangular tube? cant it be a circular tube?

Q2 (Ans = B)
Drag coefficients (3-D), refer to ellipsoid.
Assume laminar flow: L/ D=2 →C D=0.27
F D =0.5 ρV 2 A C D →V =¿
VD
Check: ℜ= =2490421 → Not laminar
v
Assume turbulent flow: C D =0.13
V =9.37 m/s

Why do you use Area of circle? Instead of an ellipsoid?


Because if you look at the object sinking towards you from the bottom, the cross section is
circular. Using 0.5*area of ellipse will give you the same answer and it is more correct.

Confirmed b) is right as A IS THE CHARACTERISTIC AREA FOR IMMERSED WETTED


AREA
Q3: (Ans = C) Agree: 7, (Ans = D) Agree: 3
To help us understand:
In this question’s case, since β1 = β2 = 90°, Vt1 = u1 and Vt2 = u2, no? If this is correct, result
should be http://www.wolframalpha.com/input/?i=998E-4%28100pi
%2F3%29^2%28.04^2-.015^2%29

What would reveal this question to be quite misleading, since it uses the symbol ω to refer
to frequency, while the equation provided in the formula sheet uses the same symbol for a
different thing (angular velocity).

In the question, where it says, the water enters and exits parallel to the impeller blades, is
that referring to the relative velocities, w1 and w2? I think so: the figure seems to be just a
way of helping students better visualize the pump’s geometry... Yeah that’s all good. It
makes sense though, because if the w1 and w2 are parallel, then the Vt1 = u1, which other
people have said.

P=ω T =ω ρ Q(r 2 V t 2−r 1 V t 1 )=ρ Q(V t 22−V t 12)


V t 1 =ω r 1=1.57
V t 2 =ω r 2=4.18 Make sure you convert RPM to rad/s
P ≈ 1.5W

So Power = T*w = 1.504 * 104.72 = 157W (d)+3 -1 << THey have already converted the power
equation from P=rhoQ(u2*Vt2 - u1Vt1) to P = rho*Q*(u22-u12) by recognising that u2=Vt2 and
u1=Vt1, I get 1.504 as well.

Q4 (Ans = B)
4A
Hydraulic diameter: D H = =0.08 m Where A=area, P=perimeter
P
V DH
ℜ= =20378.11 v=1.005 ×1 0−6
v
ϵ /D=0.0004 /0.08=0.005
Sub all values into Haalands fo formula: f =0.0342
User energy equation for flow, where V 1=V 2 , z 1=z 2
ΔP L V2
=h f =f → Δ P=13.98 Pa/ m Where L=1m, as it ask for per unit length
ρg D 2g

Q5 (Ans = A) Agree: 10
L= A /b=1.4 m
VL
ℜ= =4.67× 1 06
v
AR=b 2 / A=5
From NACA0009 Chart: C D ,∞=0.009
C L2
C D =C D , ∞ + =0.05 Note: round up to 0.05
π ARe
F D =0.5 ρV 2 A C D =735 N

Q6 (Ans = B) Agree: 7 (Ans = D) Agree: 4


Why wouldn’t it be a normal shock to trip it down to subsonic before the area expansion
increases the Mach number further.
I think letter b actually refers to the phenomenon (agreed * (5) * THE question asks for the
waves that or-iginate from the corners, so the correct answer should be letter d… (x4 agree)
^ I wouldn’t read too much into the question... goodness knows they could do a better job
of wording the questions anyway. It’s true that the expansion fan consists of lots of mach
waves, but if you just answer mach wave, that could imply a single mach wave, which is
pretty weak and doesn’t do much on its own (http://en.wikipedia.org/wiki/Mach_wave). I
think they’re looking specifically for the expansion fan effect around the sharp corner It
says “Mach waves”, plural - and therefore the answer has to be (d)
Wouldn’t it just be a normal shock wave? © definitely not
Is it that several mach waves form an expansion fan? The question asks what types of
waves originate from the corners and therefore the answer is mach waves. Thoughts?
Doesn’t the waves plural just refer to the two corners? So two single “Mach waves”, emerge
from the corners, making the correct answer b - expansion fans

Q7 (Ans = B) Agree: 14
V 1=Ma=M × √❑
Table B.2
M 1=4 →V 1 /V 2=4.5714 →V 2 =303 m/s
For laboratory reference frame: V =V 1−V 2=1081 m/s

Q8 (Ans = A) Agree: ?? (Ans = C) Agree: 54


With respect to letter B:
Friction decreases stagnation pressure in all cases. Friction increases speed whilst
subsonic due to pressure drop following the drop of stag. Pressure and hence an increase
in velocity (Bernoulli's). This is opposite in supersonic flow, although stag. Pressure still
drops, the Mach number decrease dominates following the complicated relationship and he
nce drops speed.
With respect to letter d

about a:
Doesnt heat addition change stagnation values? Or is that only stagnation temp? If it is
increased by heat additon A would be incorrect

Answer: B wrong isn't it C? P0 decreases for supersonic and subsonic.


http://books.google.com.au/books?id=YK-
jXgrSYVEC&pg=PA270&dq=Friction+acts+to+decrease+stagnation+pressure+for+subsonic
+flows&hl=en&sa=X&ei=OlpfVPP9HIa3mgWWmICQBA&ved=0CB4Q6AEwAA#v=onepage&q
=Friction%20acts%20to%20decrease%20stagnation%20pressure%20for%20subsonic
%20flows&f=false

I disagree, maybe you read the question wrong ? I read it as which are true. A seems true -
is the definition of stagnation pressure. A is only true for incompressible flow
(Bernoulli’s Equation), i think A is false. Red, you need to redo 1500 cunt

Also, B is false - stagnation pressure is like the total momentum - either stored in pressure
or as kinetic energy, so friction will always decrease it.+1 Alternatively you could look at the
friction tables and see that the quantity p0/po* is always greater than 1. C is also true.

OPtion C is also true, also option c is true, by the way option c is correct -
Cooling will reduce the stagnation temperature and so will decrease stagnation pressure.
SO my final answer was A, C and D. (agree x1) C is NOT the cooling alternative. The
alternative says that the stagnation pressure CANNOT be increased by cooling flow. Since
cooling CAN increase the stagnation pressure, alternative D is FALSE. I’m very confident
that B is TRUE.

“Cooling can increase stagnation pressure” - Ingo


Cooling can increase stagnation pressure -Vince
D is false. Hey, did they elaborate on how cooling can increase stag
pressure? Check the slide. It says opposite for cooling. So D is wrong, it
does not ALWAYS :)
Part B
Question 9
Back pressure: pressure at the back of the exhaust (i.e. atmospheric pressure)
A) (Ans = 368.9 kg/s, Choked) Agree: 18
Summary: pb/p0 = 0.02 < 0.5283, so flow is choked.
Mass flow rate most quickly calculated using the choked mass flow rate equation on the
first page of the compressible flow equations. mdot = 369 kg/s (+9)
(James) p0/pb = 50, so it’s definitely choked. what p0/pb would mean its not choked??
Sorry, a better way to put it is that pb / p0 = 1/50 = 0.02, so looking at table B1, p/p0 for mach
1 is = 0.5283, so it would be choked. sweet thanks. Yeah, so adding to that, p/p0 at mach 1
is 0.5283, so you would expect choking at (1/0.5283) *pb = 1.893*pb. So for most of the
examples, pb = 100kPa, you would get choked flow for a container pressure of 189.3kPa
(and above)
Calculate properties at the throat (*) to find mass flow rate
P*=0.5283P0 (M=1) = 0.5283*5000 = 2641.5 kPa
T*=0.8333T0 = 0.8333*3000K = 2499.9 K
Ideal gas law to find density at the throat:
rho*= P* / (T*R) = 2 641 500 / (287*2499.9) = 3.682 kg/m^3
The flow is at M=1, so velocity is V = 1 *sqrt(k*R*T) = sqrt(1.4*287*2499.9) = 1002 m /s
mass flow = rho*V*At = 3.682*1002*0.1 =368.9kg/s

b) If there’s a shock at the exit plane, then the flow must sonically expand perfectly to
the exit plane. Please be nice to each other
Ae/At = 10.7 - Looking at table 1, this corresponds to Mae1 = 4.0 (almost)
Looking at the shock table, the exit flow will have Mae2 = 0.4350; from the same table, the
stagnation pressure over the shock drops by p02/p01 = 0.1388. The flow up to this point
was isentropic, so the stagnation pressure before the shock was 5 MPa. The stagnation
pressure after the shock is p0e=0.1388*5000 = 694kPa

The flow exits the shock at Ma = 0.435. Using table one, the static-stagnation pressure ratio
is p/p0 = 0.8780, so the static pressure after the shock is pe = 606 kPa. - How did you get
606kPA using that? With 690*0.878 = 606kPa.are you joking? you got 694k previously but
now you using 690k? The stagnation pressure has dropped over the shock due to the
stagnation pressure ratio, p02/p01 = 0.1388, giving the new stagnation pressure above. That
pressure ratio p/p0 is from table 1 and relates the stagnation pressure (which has dropped
because of the shock) to the static pressure at that mach speed.

The question tells you that this is how you calculate the minimum pressure to see if there’s
a shock in the exit plane, so you don’t need to have any prior knowledge to conclude that
there will not be a shock inside the nozzle, since the back pressure is less than the
minimum pressure causing a shock. Also, part D (and the diagram) implies that either an
oblique shock or an expansion fan sits outside the exit anyway.

i did it the opposite way using table B1 (p/p0) to get exit pressure then using B2 (p2/p1) o
find min pb for shock. got 610.4kpa which is pretty close (agree+4)< dunno what red is on
about
-- Can you please explain this a bit clearer in steps? Im struggling. So is the back
pressure the 33 or 610.5??? B1: Pe1/Po = 0.0066, then Pe1 = 33kpa. We know Me1 =
4, so we know all the properties of the normal shock. B2: Pe2/Pe1 = 18.5, therefore Pe2 =
18.5 * 33 = 610.5 kPa I understand that you get the 33kPa that makes sense and is just like
Tute 8. HOWEVER in tute 8 they leave it there without considering after the shock, saying
that if the back pressure is that value, the 33kPa then the shock occurs at the end. In our
case the pressure gr.eater therefore shouldnt a shock occur sooner?
I think what it is, is that if you have a shock at the end of the nozzle, the static pressure after
the shock would be something (in this case 606) but if you moved the shock further into the
nozzle, the static pressure at the end would increase. You could also infer from the diagram
that the this is the case because they have oblique shocks drawn in there.
Hmm, still a bit confused, I won’t lie, do you think tute 8 was correct? or incorrect. Based on
the way this problem was solved it would seem you need to determine the pressure after
the shock and compare it to the back pressure, but in tute 8 they did not. This method
makes more sense I’ll admit.

The lower curves represent lower back pressure, this is from the last slides.
Option F means that there is a shock at the exit plane. This will compress the flow to a
certain amount. If the back pressure is greater than that , options D and E, then the shock
will be further inside the nozzle, so the shock in the exit plane is the case that has the
minimum back pressure of all the cases where there is a shock in the nozzle.

If the flow expands perfectly without a shock, then it will be really really fast and the
pressure will be low. If this happens to be the exact back pressure then that's all that will
happen, and that is option H.
But if the pressure is lower than the shock at the exit plane case (meaning it compresses it
too much and raises its pressure too much) , but the pressure is higher than what’s allowed
for perfect expansion, then we have case G. A shock in the nozzle can’t get in to back
pressure conditions , so oblique shocks outside the nozzle have to do it - that is line G,
which is this question. Line ‘I’ means the back pressure is even lower than for perfect
expansion and mach waves (like an expansion fan or something) will have to make the flow
accelerate and therefore lose even more pressure.

C) (Ans = 719kN) Agree: 9


From part b, the flow expands to mach 4.0. There are no shocks inside the nozzle.
Table B.1
M 1=4
T /T 0=0.2381→ T e =714.3 K
P/ P0=0.0066→ Pe =33000 Pa
V e =Ma=M × √ ❑
F=719 kN Where Ae =1.07 , Pb =100000 Paand mass flow rate is from part a.

Why use the same mass flow rate?


Because Pb is required for choke flow which then gives us the mass flow rate (see (a) for
full description). the question states to use Pb = 100kPa hence the use of the same mass
flow rate. You would assume that the conditions are ‘steady state’ so the mass flow rate is
the same as measured at any point.
Is it even right to use M=4? Because we found that the back pressure had to be 610kPa to
achieve this?
The fluid goes up to the back pressure by hitting the oblique shocks. The formula wants
pressure at the exit plane, if it passes through shocks later, it doesn't care. M=4 is what the
speed would be if the flow expanded in the nozzle without a shock, the oblique shocks
happening outside the nozzle are what will cause the static pressure to correct itself to the
static pressure.

Q9D is non-assessable according to Vince.


The pressure at the exit plane was pe = 33kPa, so it needs to be compressed up to the back
pressure which is pb = 100kPa.
Oblique shocks increase pressure, (expansion fans do the opposite). So the shock has a
pressure ratio of pb/pe = 3.03. You look through the normal shock table to see that this
corresponds to a normal shock at Ma = 2.26. {These numbers were read off the table wrong,
but the method is right - read bright green guy below} This is the mach number of the
normal component of the flow (normal to the oblique shock), so Man1 = 2.26.
Man1 = Ma1 sin (beta), therefore beta = inv sin(2.26/4) = 34 degrees.
THen use the oblique shock table, mach 4 is the right side of the axis and at beta = 34, the
angle theta is about 23 degrees.
i think you looked up p02/p01 instead of p2/p1 (p01 = 5mpa not 33kpa) i get man1 = 1.7, beta
=25.15 deg, theta = 12.5 deg Yes, you’re right, I was looking at the density ratio instead.
Could also just use p2/p1=3, M=4 and read both angles directly off figure B2 yeah? Seems
like several ways to do it. I would back this way, it’s pretty easy and gives theta ~13 and
beta ~25

The part about it reflecting - I guess you would need some reflected oblique shocks - after
the oblique shock, the flow from the top and bottom half are moving towards each other, so
they must be deflected again away from each other, or parallel to each other. The flow after
the oblique shock is still supersonic (from the chart used to find theta, it says that the total
mach number Ma2 is greater than 2), and I think the only way to really turn a flow again
would be with oblique shocks ?
Question 10
a) i) Axial velocity: Va = Q/A = 80/1.6=50m/s

This is what I got not sure if it is correct. Don’t know how to solve the exit angle
I’m pretty sure your U1 is wrong. It points into the tail of V1, so you need to move it up to
where Vt1 is sitting. Check part 2 of turbomachinery lectures. Also, your rotor blade triangle
is wrong too, V1 should be W1 (as your rotor spins). You want your V1 (rotor) to be in the
same direction as Vn1 so that when you include another stator the incoming flow will
cause no shocks and enter parallel to the blades again.
ii)
Like this, but flipped the other way. The important thing is that beta1 is the angle that the
relative velocity, w, enters the stator ( ‘shockless’). Also, u and w act in the same tangential
direction. Also mark Vt1 and Vr1. The diagram above marks beta relative to the y-axis, but it
should be relative to the x. (these comments refer to the first diagram, not the hand-drawn
one)

III)vt = vr tan (70) - u1 = 7.31 m/s Shouldn't this be +*u1?? instead of - u1?anyone explain?
For some reason, I get this as = 6.47m/s. Did you get u1 = 130.9?, Vt1 is the quantity,
Vr1*tan(alpha) = 137.37m/s . If you look at the diagram above, the tangential component is
bigger than the which is the first quantity in your calculation radial component, so 137.7
makes more sense than 7.31m/s, I think . What is u1 and how do you get it? u1 is the
blades rotational speed and it’s found by: u1=wr(av), so for this case,
u1=261.8*0.5=130.9m/s.

IN THIS PROBLEM ALPHA IS MEASURED RELATIVE TO W NOT V CORRECT?


Alpha is the angle between the stator trailing edge and the horizontal. This means it’s also
the angle between V1 and the horizontal.

beta1 = atan(vt/vr) = 8.4deg I agree with the answer, but the numerator is not Vt1, but (Vt1 -
u). it might just be because we defined vt differently Yeah but it is important for
calculating torque and power later on. The way it’s defined here, vt1 is actually the
tangential component of the w1 vector, but it should be the tangential component of the
absolute vector, V1. if you draw it properly our vector diagram is different to that of above
as our alpha is larger than beta. so with my diagram i have found vt1 as the tangential
component of v1 not w1… Wouldn’t tangential velocity, Vt1 = Vr1tan(70) ? nvm i fucked up
my what i called w and v
tan (beta1) = wt1/wa1 = (Vt1 - u)/Va = (Vr1 tan(70) - omega*r) / Vr1
So beta1 = atan((50tan70 - (0.5 * 2500*2pi/60))/50) = 7.37….. not 8.4? Why? < I also got 7.37
as my final answer. panic mode in exam (agreex200000)
ANYONE GOT 8.4??
yes i got 8.4
I got 8.12..close enough :D
You can get 8.4 if you use shitty rounding for u: working backwards -> tan(8.4) =tan
(alpha1)-u/Va -> u=(tan(70) - tan(8.4))*50 = 129.99 m/s which chump rounds 130.9 to 130?
Did anyone get 8.4 without using shitty rounding? I can only get 7.38
yeah same
Exam typo? Should it be 7.4? Yes fuck oath(I’m with you)

iv) b2 = atan(u/vr) = 68.96 deg (agree x8)


I got this as well. < i got 79.187 deg using whole number atan(2500*2pi/60/50)=79.187 deg
<<Multiply your numerator by 0.5 ( r ). At the moment, you have atan (Omega/Vr)>> << got it!
(x2)
Are you guys rounding u to 130m/s rather than 130.9? Yes < please no! yes.
If use 130.9m/s the answer is 69.1deg

b i) roh =P/RT =5.80 kg/M^3


t = roh Q(r2*vt2 - r1*vt1)
r2*vt2= 0
found vt1 above,
t = 1711.9nm
I get 31.984kN.m (agreex9 1 but approximate 31.87) IS there any working for this? I get the
same as green well, since we found rho as 5.81, just sub it in the eq T=5.81*80*0.5*137.37;
T= 31.95kN.m ah yeah green just used the relative tangential velocity
why do we have to consider it’s swirl free (Vt2 = 0)? Cause we designed earlier (part a iv) )
to have swirl free exit
to the blue and green one : vt1=va*tan(alpha) we dont need minus u part… right?
ii) p = omega * t
p =448 kw
I I got 8.373 MW (Agree X9 but approximate 8.29) - IS there
any working for this? I get the same as green There’s some discussion above, explaining
that green got this by making a little mistake in defining what Vt is. Cheers
Just wondering, according to the equations for torque and power, the answers should end
up being negative, but does that just tell you that power is being extracted from the flow?

c. a= 694.377.
so, ma = .072
dont need to consider compressible effects?? I don’t think so, since 146.2/√(1.4*287*1200) ≈
0.21, below the threshold usually used for compressibility consideration.
I think that for axial turbines/pumps, only a small amount of work is done on the rotor, and
so there are relatively small pressure differences. A real turbine will have many stages,
each with small pressure differences. So over a single stage in the turbine the pressure
difference and compressibility is negligible. You might have to think about how at later
stages, the density is actually different, but again, over a single stage it shouldn’t matter.
You could maybe apply the bernoulli integral over the system, and show the what the static
pressure change is, work out how much the density will change and show that it isn’t much.
Also, from the problem statement: “(For the calculations you may assume that the flow is
incompressible)”Any ideas?
Question 11
a)i)v= sqrt([z0-z2+hp]/[2g{1+f Lt/D + sumK}]) where Lt = L1+L2 , D =D1 =D2

What f did you guys use?

This is what I found: vel=√ ❑

Why can you assume that V2 = Vmean ? Becasue D1=D2? Mean velocity is the average
velocity of the entire pipe. hence the Vmean is applicable throughout the entire pipe.
Doesnt the V2 term get removed because you sub in v2^2/2g as -20. at the exit the head is
all dynamic head from the pump. otherwise you would have a 1 floating in the equation
somewhere. when v1=v2, the v term is cancel out (v1 - v1 = 0). which it only leaves the v^2 /
2g term in hf and hm. the -20 is the head of pipe which is hpump. from here on, derive the
v. wont V in the part before the pump be less than v in the part after the pump, because the
pump speeds up the flow??????????????????

this is just what i found (agreed * 4)


Are people agreeing that we just assume f is the same for both pipes? Or are we expected
to make the equation in terms of both f1 and f2? I dont think this eqn is right, i agree with
vel=√❑, v1 cannot cancel out with v2 because v1=0 and v2 is not.
Zo is at top of reserviour therefore v1=0
Not true^.You can cancel because the areas are
the same, and the flow rates must be the same therefore
the velocities are the same and can be cancelled.

It is not stagnant at point 2 hence the extra 1 in the denominator. Otherwise correct after

replacing ∑ K with 9.22 in terms of the function it was told to express. (agreed x3)

Q1=Q2 -> v1=(d2/d1)ˆ2*v2 , therefore being 2 diffent f. In week4 tutorial Q2 is similar.

ii) v =.22m/s Q=.007m^3/s ( i think this is wrong)


I think you would have to consider that the reynolds number in the two different pipe
sections is different, and so the friction factor would be different too. I just noticed that the
diameters are the same, so actually the friction factors would be the same. -This also means
Re will be the same

Using different f1 and f2, I got v=4.724 and Q= 0.14 m3/s [..] Got the same ←-you forgot to add
in sum of K; Got v=5.3867, Q=0.169 same +1. Why would you use 2 different friction factors?
The diameter and therefore the velocity is the same throughout. Also I thought the point of
using the mean velocity is so you can just use a single friction factor. No idea how this
whole thing would work if the 2 diameters were different though. I believe it would turn into
a shitstorm. Yeah i wrote that there may be 2 f’s before realising the diameters were equal -
so only one friction factor is involved. @yellow and red, I realize that the diameter of the
pipes were the same, but notice the length L1 and L2 are different, so, when you solve it,
you will get different ratio of f. it will look something like this :

L1/D*f1 + L2/D*f2= 500*f1+250*f2 <- Yeah sorry this doesn’t prove f1=/=f2, just that friction is
experienced differently in the different pipes, which is expected.

hence, assumptions of f1 = f2 = f is not valid. you will get 2 different reynold number using
same roughness ratio. iterate twice to get to the final answers (2 hours exam, GG) -
reynolds number is based on diameter anyways in pipe flow… Agreed, problem is
simplified by assuming same f factor. If it were otherwise you would be implying different
velocities or different roughness. Since (1) we’re only given one roughness ϵ or (2) would
need to do bernoulli twice to get a velocity for two different sections. Either of these would
overcomplicate it and make the problem unsolvable with the information given (eg. we don’t
have Z1).
My results were (agreed *9):
● ∑K = 9.22 (do you include submerged outlet?) - no, because the outlet is a free-
flowing hose end. They included some extraneous information in that table, like the
fully open/ closed valve. Cheers!
● ε/D = 0.025*10-3 / 0.2 = 125 μ
● velocity ≈ 3.5 m/s (5.49m/s is the correct ans with f = 0.031725)
● flow rate ≈ 0.11 m3/s (0.1725 m3/s is the correct ans)
● what f value did u end up with? Got 0.03615234021 Same(2)

(b) any idea of this? I assume Vin = Vout, so Dout=0.0734m. not sure about this.

Not sure if it is right… but the way I understand it is:


● the adding of the nozzle will make the pressure at cross-sectional plane 2 not
atmospheric anymore; (can you explain why?) P2 would increase as the diameter is
reduced, resulting in pressure buildup at the nozzle.Thus the water will flow out
faster.
● the new velocity inside the pipe is ≈ 0.64 m/s, that leads to a Reynolds number of ≈ 1,071(used
L=D in reynolds eqn), that in turn points to a friction factor of ≈ 0.06;
● then, the gauge pressure at cross-sectional plane 2 is ≈ 186,259 Pa; (how did you get
this?)?????????! help here on P2. seriously
how did you get this value?
cause you still have two unknowns in the head equation (p2 and
v2). i managed to get 0.035 if I let P2 = P1, however not sure if this
is right?

● next, the velocity at the nozzle’s exit is ≈ 20.7 m/s;


● finally, the nozzle’s exit diametre is ≈ 84 mm (I was wrong: please see below). I agree,
sounds legit. So you end up converting the pressure head inside the nozzle to
velocity head outside? Only thing is with that last step I ended up with 35mm. Did
you use Q = 0.02 and find the area using Q=VA then convert to D from there?
● can someone explain this in more detail?
● D=0.035m (calculated from v=20.7m/s) I agree x2 How did you get 20.7m/s? I thought
we found V to be 3.5m/s from the last part?

Wait, what the fuck is it saying, like how can it limit the flow rate, does not the same amount
fo water enter the pipes and thus the same amount has to exit it???? Continuity shit
yeah----Yeah i also don't understand this, doesn't continuity have to hold and therefore Q
must remain constant? how can a simple area change alter the flow rate? someone please
explain +1+1+(>9000)
Not 100% sure but I think adding the nozzle brings P2 back into the flow energy equation
since there is pressure inside the nozzle. Before it was atm so cancelled out with P1. So
that results in a lower dynamic head to compensate for the higher pressure head. Then that
pressure needs to be converted back to dynamic head outside the nozzle to get back to
zero gauge pressure. Once you find that velocity then you use continuity to find the nozzle
area. So Q does remain constant, its just a lower Q than before.

Guys I think you have wayyyy over complicated this

Can someone pls explain how to get the pressure at P2 or the velocity V3?? I understand
the theory behind it all, just don’t know how to get the values??

apply bernoulli’s across nozzle to find P2 (=Pinlet) = Pout c+0.5*rho*V^2*( (D2^4 / Dout^4) -
1)
(Pout = Patm) Take energy equation again, except with new P2 → solve for Dout
What V^2 term is used here?^
a
Yeah applying bernoulli’s across the nozzle gives you the pressure that goes into the
nozzle. then you go back to your original energy equation for the rest of the pipe system,
except this time you keep the p2 term in and solve for that, then set p2 = p_(inlet of nozzle).
The velocity terms will give you the diameter needed, after applying Q=VA. I also got 35 mm
+1

c)
i)
CQ1=CQ2
D2/D1= 0.7937 (agree x-7) + 1
HELP! How is this done?? D1 = 0.2? D2 = ?
Cq = Q / nD^3 -> Cq1 = Cq2 = Q1 / n1*D1^3 = Q2 / n2*D2^3 where n is the RPM of the
pumps. The RPM is converted to rad/s. from this eqn find D2.
I don’t believe it’s necessary to convert to rad/s since RPM/RPM is dimensionless
-it’s not dimensionless. rev/min is a dimensioned unit #idiot

ii) Ch1=Ch2
H2=50.396 (agree x7) +1 x bad boi
iii) higher head> higher velocity> higher reynold> lower friction factor> complete
turbulence> good! (halp!) In (i), didn’t we say Q1=Q2? Therefore, the velocity would
have to stay the same? So the only thing that can increase would be the nozzle
pressure (friction factor can’t increase because velocity won’t increase). But then I
don’t get what they mean by “how would you adjust the system to return to the
target flow rate”, since it’s already at 0.02m^3/s. Higher head = More turbulence, the
question is asking how would we adjust the system using the higher rpm pump. Q =
VA, higher velocity leads to higher flow rate, so to reduce the flow rate would be to
reduce the velocity. Looking back at the equation in part a) i), to compensate for the
higher head, i think the reasonable adjustment to the system would be to 1) decrease
elevation between tank surface & pipe outlet 2) increase length and decrease
diameter of pipe 3) intro more elbows and bends (more loss coef) 4) rougher pipes.
jjj) How about you just close the valve a bit more which will introduce greater minor
losses and lower the velocity back to the desired rate. (JP) ← this guy
kkk) Also, according to part b) (and a)) the pressure at the inlet to the nozzle (P2) would
increase because the hp (pump head) is now higher, therefore, looking next at the
equation used to calculate D3 (diameter of the nozzle outlet), to compensate for this,
D3 would now have to be decreased. Is this valid reasoning?
Question 12
a) (James) @M1 = 0.1, A1/A* = 5.8218,
A2/A* = A2/A1 * A1/A* = (0.0102/0.05) * 5.8218 = 1.1876, so M2 = 0.60 (agree x9) why is it
subsonic and not supersonic? shouldn’t the M>1 after it pass the choke point? No, because
it’s only contracted. And it wasn’t a choke point either. Yeah a throat isn’t necessarily
choked. have you been to a lecture like all us other gimps? Nah

@M1 = 0.1, p1/p0 = 0.9930 and @ M2 = 0.60, p2/p0 = 0.7840,


p2/p1 = p2/p0 * p0/p1 = 0.7895 (agree x7)
@M1 = 0.1, T1/T0 = 0.9980; T1 = 300, so T01 = T02 = 300.60K (agree x7)

b) A2 is given, so find that D = 0.11396 m


f (delta)L/D = 0.02*1/0.11396 = 0.1755
@M2 = 0.60, (fL*/D) = 0.4908
f (delta)L/D = (fL*/D)2 - (fL*/D)3
0.1755 = 0.4908 - (fL*/D)3
(fL*/D)3 = 0.3153
---->(Dont you think the Mach number should be in between 2.0 and 2.1???) Nah, friction
always drives the flow toward M =1, whichever side you’re coming from.
Interpolate
@M2 = 0.60, p2/p* = 1.7634 and @ M3 = 0.6623, p3/p* = 1.5961
p3/p2 = (p3/p*) * (p*/p2) = 0.90451 (agree x5) can anyone explain how do you guys get
M3=0.6623? Interpolate between .6 and .7! Your fl/d lies in this range. Table b3. I suppose u
assume its not supersonic so that it can turn supersonic aft the heating?? Dk. Cool, got it!
thanks

c) The maximum heat without minimising flow rate will be where state 4, T04 is the
critical stagnation temperature that causes choking (T04 = T0*). More heat can be
added but the mass flow rate would have to drop. Also, the friction flow process in
part B was adiabatic, so the stagnation temperature at point 3, is the same as at
point 1 (and was calculated in part a) Could someone please explain how the mass
flow rate is affected by heat transfer? How does it change when you are in subsonic
flow or in supersonic flow? I’m still not sure about what the question is exactly
asking...would really appreciate any help.
The incoming flow is M3 = 0.662, T03/T0* = 0.8745. T03 = 300.6, so T0*=T04= 343.76K
The heat is q = cp(T04-T03) = 1000 *(343.76-300.6) = 43.16 kJ/kg (agree x7)
To maximise heat input, the flow is choked, so M4 = 1.0 (agree x4)
@M3 = 0.662, p3/p* = 1.4889. Since state 4 is the critical state, p4=p*, so p3/p4 = 1.4889. It
actually asks you for p4/p3 = 0.67164 (agree x5) How is p3/p* 1.4889 when we got 1.5961
from the previous bit? That was for adiabatic flow. For this part we must look at the table
with Heat transfer, as it is not adiabatic anymore.
This seems like not too much work for a 7 mark question, so maybe this is wrong.
This seems reasonable to me. think the main purpose is to test if we realise what choking
does/means/how to do it with the * values. Not much work at all, took me half a page
None of these questions are hard. all they test are the key concepts (such as as
understanding the tables)
d) The flow leaving the preheater must have a stagnation pressure of 5MPa. The
amount of heat put into it was maxmised, meaning it is choked and has M4 = 1.0. So
from table B1, p4/p0 = 0.5283, so p4 = 2641.5 kPa. All the pressure ratios were
calculated throughout the question.
p1 = (p1/p2) *(p2/p3) * (p3/p4) *p4 = 2641.5 * (1/0.7895) * (1/0.9051) * (1/0.6716) = 5504 kPa
(agree x6)
2012
Part A
1. c DCCCCCC ← why? , A
Assuming “quantified in the same way” means they all have the same confidence
intervalSign in
poorly worded question
Actually it is A (Fact), since some estimators are biased, but increasing n for these
estimators will reduce the bias. However I doubt that they thought about this, when they
made the question. I think bias are errors caused by reading method? so no matter how
many times u do it, it’s wrong
Definitely not A, bias error is when each reading you take is to the left an amount, no matter
3how many times you take it that will not change. this can be fixed by doing analysis in
seperate labs/different equipment <- agreed. Bias error is an underlying error in the
experiment that cant be determined by repeating. An example would be a sensor that
measures nonlinearly at some value and was linear for all other values. This might give a
result that looks consistently parabolic, whereas the real phenomena is linear. Therefore
testing over and over would yield the same result and would not remove the bias. You need
to test with a different setup to remove it or have knowledge of the cause of the bias.
You guys are talking about one particular kind of bias (Systematic error). It is true that this
kind of bias can not be minimized by doing more experiments. But if the bias comes from
the way that you are estimating something, for example the variance as 1/N*sum(squared
error), then you will have a bias, which gets smaller, as you increase N.
he referred to bias error as above^^ in the revision lecture. so i’ll trust that
So the answer is A?
2.
Answer is C
P1/rho*(n1^3*D1^5) = P2/(rho*n2^3*D2^5)
n2=½*n1 and D2=2*D1
Sub in values, you get:
P1/(rho*n1^3*D1^5) = P2/(4*rho*n1^3*D1^5)
P1=P2/4 therefore P2=4*P1=200kW

3. b
smooth walled, ε/D = 0
L = 0.4 ← what? i thought L=400m
A = 0.01*0.005 = 5E-5m^3
Q = 30/1000/3600 = 8.3E-6m^3/s
V = Q/A = 0.167m/s
Re = ρVL/μ = 998*0.167*0.4/1.005E-6 = 66.3E6 (turbulent)
It’s only luck that this worked out, used kinematic viscosity (nu not mu), L = 400
Re = VL/ν = 0.167*400/1.005E-6=66.47E6
Hydraulic Diameter D_H= 0.0066 m D_H = 4A/P for anyone who forgot whats P? P is the
wetted perimeter. So 0.03m in this case since we assume the channel is full
Re (D_H) = 0.1667*0.0066/(1.005E-6) = 1095 (Laminar)
f_lam = 64 / Re(D_H) = 0.0579 = 0.058

When do we use D_H for Re and when do we use L? SOMEONE PLEASE


ANSWER!!!!!!!!!!!!!!!!!!!!!!!!!!!!!!!!!!!!!!!!!!!!!!!!!!!!!!!!!!!!!!!!!!!!!!!!!!!!!!!!!!
When you have non-circular pipes you should use D_H
Use whichever is relevant. For internal flow, use diameter. For external flow, where there
isn’t a diameter, use length. For external flow across a wing, use chord length.

Haaland gives f = 0.063 - yeah using Re=1105 in haaland gives this. Why can’t we use
Haaland in this case? it’s laminar
so Haaland only apply to turbulent flow?
According to Ingo: “you use 64/Re is the flow is laminar and Haalands formula when
flow is turbulent.”

?Why not right?1/(f^0.5)=-1.8log(6.9/(6.61e7)....leads to friction f=0.0063(none of the


anwser…)
ANSWER TO ALL YOU SCRUBS(+1): There are two types of turbulence. 1) caused
by flow instabilities which is to do with the cross section and 2) to do with
instabilities due to wall friction. If its smooth walled (it is in this case) then you
automatically know that the turbulence its refering to is due to cross
se+6+6+6+6+6+6+6+6+6+6

4. A
using coeff of drag= Fd/(0.5*density*vel^2*A)
where vel= 50m/s. So CD=1.1772

5. A +4
Re=0.2*5/(1.5Ε-5)=66.7Ε3, turbulent <<incorrect, as Laminar (<5x10^5)
Displacement thickness at x=5:
δ*=5*0.02/(66.7E3)^(1/7)=0.0205m

Find Re_x at 5 m = Ux/ν = 0.2*5/1.5e-5 = 66.667e3 which means the flow is LAMINAR.
Why its not turbulent?
(transitional Re for flat plate conditions is 5E5, p12 of the formula sheet)
Such that the displacement thickness (δ*) is 5 * 1.83 / (Re_x)^0.5 = 0.0348 m
^Momentum Integral estimate for displacement thickness (δ*) for laminar flow from the
formula sheet. - Why do you use laminar not Blasius? - Question says to use momentum
integral estimate. Kk thanks

Using conservation of mass, velocity at 5 meters = 0.2316 m/s = 0.232 m/s = A


how did you get 0.232 from (δ*) = 0.0348 m???? its in the week 5 tut solutions

The above comment in blue… does this account for the boundary layer inside a pipe can be
approximated to a flat plate boundary layer???
^Yes. Thats exactly how the problem has been solved. The δ* is for a flat plate.

Hey, cons of mass meaning ? I get 0.1335m/s using that :/


Try mass flow rate in = mass flow rate out.
such that ρ*V_in*A = ρ*V_out*A_out.
Think about a square tube, where the side length is reduced by 2 δ* (boundary layer inward
on both sides).
ooooooh that makes sense, ta! (boss)
*you can also refer to page 19 [external flow part 1]

In case anyone was confused with how this answer came out, once you get δ* , and use
conservation of mass, it becomes VinAin = VoutAout => 0.2 * (1^2) = Vout * (1 - 2δ*)^2….
solve for Vout. The area of the flow out becomes the original area (1m^2) minus the
boundary layer of the top and bottom of the place.

Still confused with why it is used the area of a square Purple has used a circular duct, they
just didn’t put in pi since it cancels anyway. And you have to use 2δ* since the layer is
reduced the whole way around. The blue circle is applying δ* and you can see the purple
goes around the whole circle. If you were to just measure the diameter, δ* is on the left and
the right hand side, meaning 2δ*
.

6. a
ω(Μ1=3)=49.76, so flow is turned 45°(parallel) and the rest of the momentum goes into
increasing the mach number
Not sure about this next bit but maybe someone can confirm:
M2=M1 + M(ω=49.76-45=4.76) =3+1.25 = 4.25
I think you can use the formula: P/P1 = (1 + (k-1)/2*M^2)^(k/k-1) (IS THIS EQUATION GIVEN
OR MUST WE MEMORIZE? IS THERE NO OTHER WAY TO SOLVE?) to find the pressure
ratio. If the ratio is > 1 the nozzle will be underexpanded (expansion fan with flow turned
outward), if the ratio is = 1 the nozzle will be ideal, and if the ratio is < 1 the nozzle will be
overexpanded (oblique shock with flow turned inward). So from this i think the answer is
(a). Refer to Oblique shock lecture, slide 9. yeah but pressures aren’t given dawg
M2 = 7.8
M2 = 7.8
M2 = 7.795454545454545
ω=45+ω(M1=3)=45+49.76=94.76
M2=M(ω=94.76)=7.8
Also, not too confident about what I said above anymore:
From the wiki page:
“If there is not enough pressure to start with, the flow won't be able to complete the turn
and will not be parallel to the wall. This shows up as the maximum angle through which a
flow can turn. The lower the Mach number is to start with (i.e. small M1), the greater the
maximum angle through which the flow can turn.”
But am not sure how to use Table B.5 to determine this max angle...

7.a
4 A answers in a row. Mega-suspicious.

To help us understand, this is the textbook’s table on the effect of friction to the flow:
D=0.5m, L=1km
f=0.002
M1=0.1
fΔL/D = 4 = (fL*/D)1-(fL*/D)2
(fL*/D)2 = 66.9 - 4 = 62.9; M2~0.1 naaaah wtf I disagree with this now :(slightly le the M2~0.1
is actually ss than 0.1… which makes no sense, as it should have acellerated a little bit. No,
M2 is slightly bigger. It’s right.
If u interpolate table B3 to find M2, it should be 0.25 from what i calculated. ←lol? you using
62.9 for your (fL*/D)2 ? This is impossible to get 0.25 because the range is from 0.1 to 0.2
only. <<AGREED. I got M2 = 0.1076 from interpolation. Can anyone confirm? +1 yes I got
0.1076 as well.

Agreed on 0.1 from that method - However not sure if choked flow has something to do
with it?
+1 Agree on this method. Compressibility is negligible (and reality check, mine shafts in
reality are much wider and significantly longer lol)

Isn’t the value for fl/d=4 between .3 and .4, therefore .3 is the answer? What am I missing?
See above equation for fΔL/D = 4 = (fL*/D)1-(fL*/D)2

when we refer to table its fl/d. whats the meaning of l? delta l or l1 or l2?

8. d +3 I got the same


P1=150*101.3=15.2MPa, P2=101.3-kPa
M1=0, P1/P0=1 => P0=P1 are we using stagnation pressure formula p0/p ? so on the formula
sheet, p0 is on the top instead of bottom, please help, a bit confused! Since the answer was
1 P0/P1 is the same as P1/P0. They probably just mixed up the subscripts.
P2/P0=0.1013/15.2=6.66E-3 => M2=4
Part B
Question 9
a)
i.
Re=VL/v => L=Re.v/V=5E5*1.5E-5/V=7.5/V (why is it 7.5?)

10% of banner length 0.1L=1m


1=7.5/Vmin => Vmin=7.5m/s
ii.
assuming forces act through centroid
Σ bout the corner point, there’s gravity acting downwards at the centroid and there’s
tension acting away from the flag. 2*T and 5*80 I’m assuming because he’s used the full
length and halved it by giving twice the tension from it, it’s kind of roundabout in my way of
thinking.Wouldnt it just be: T - 10*80g though, for the full length of the banner?
The T isnt tension its shear stress
Cf=τ/(0.5ρV^2.A)=0.027/(VL/v)^(1/7) <- There shouldn’t be an ‘A’ term in this equation. < I
agreex2! <τ is force due to drag isn’t it? From 0=2τ-5*80g. It’s been divided by area to
become shear stress.
V=(1+6/7)th root(τ/(0.5*ρ*Α*0.027)*(L/v)^(1/7))= √(1960/(0.5*0.2*40*0.027)*(10/1.5E-
5)^(1/7)=√(123259)=551m/s. fast helicopter…
I got 151.6 m/s
im getting 210 also getting 210
I get 133 m/s I also got 133 (agreed x6)

Σ moments about X: 5*m*g-2*F=0 < wrong, it should be 5mg- 4F=0 (both sides)(he accounts
for this by doubling F below) can someone explain how to get this moment equation?
Assuming weight acting downward through centroid and drag force (due to skin friction)
acting to the right through the centroid as well, lever arm to weight it 5 m and lever arm to
drag is 2 m.
F=1962N
So what does the F in this equation refer to, is it shear force?
T_w=C_f*0.5*ρ*U^2 <<<< doesnt this need an area term in it?

Assume we will be traveling faster than 7.5m/s (transitional velocity calculated in part i)

C_f=0.027/Re^(1/7)

F=int(T_w*dA)
Integral is from 0-L
(dA=H*dL as the shear does not change over H)
Multiply by 2 for friction on both sides - why do we do this? flow generates shear, and flow
is on both sides of the banner

Bringing these together


F=int(2*0.5*ρ*U^2*C_f*H*dL)
Bringing constants out the front and subbing C_f
F=ρ*U^2*H*int((0.027/Re^(1/7))*dL)

F=ρ*U^2*H*0.027*int((1/(U*L/ν)^(1/7))*dL
F=ρ*U^2*H*0.027*ν^(1/7)*U^(-1/7)*int(L^(-1/7)*dL)

F=ρ*U^(13/7)*H*0.027*ν^(1/7)*int(L^(-1/7)*dL)

F=ρ*U^(13/7)*H*0.027*ν^(1/7)*[7/6*L^(6/7)] fromL=0->10

F=ρ*U^(13/7)*H*0.027*ν^(1/7)*[7/6*10^(6/7)]

Sub in constants

1962=0.2231*U^(13/7)
U=133m/s
How did you get 1962??

b)
110km/h=30.56m/s
How do you determine whether flow is turbulent i.e Cd = 0.47 or 0.2?
find transition D, seemed too small thus assume turbulent and check at the end
Assuming turbulent flow:
i.
Fd=Cd*0.5ρU^2*π*D^2/4 ←Why are we not using 4*π*r^2 for Area of sphere?-frontal area
D=√(250/0.2/0.5/0.2/30.56^2/π*4)=4.13m ^
Density of air is 1.2kg/m^386*(110/3.6)/1.5E-5 = 3.433E6, therefore turbulent flow ^
agreed at 1.686m (x11), anyone see where the 4.13m went wrong? at the density, which is
why I bolded it when I showed the proper working, he had it as 0.2
Agreed on 1.686m+1 ^
What is the critical value for this Re? ^
The external flow one, so 5*10^5. ^
All the other Re_trans numbers are lower anyway, so it didn’t matter which, it’s above all.
> D=√(250/(0.2*0.5*1.2*(110/3.6)^2*π/4))=1.686m ^
Check assumption -> Re = 1.6

Choosing the relevant dimension for Re: This depends very much on the application.
Reynolds number is defined as the ratio of inertia forces to viscous forces.
● Internal Flow: Here most applications consider the flow downstream of the entry
region. Here the velocity profile no longer varies with axial position along the pipe.
However the velocity profile is a function of the pipe diameter. Thus using D as the
characteristic dimension is appropriate. (Hydraulic diameter is an extension of this
concept, to account for non regular cross-sections). Use Re_trans = 2300.
● External Boundary Layer (BL) flow: Here the dimension x from the leading edge is of
importance, as it describes what type of BL exists. Use Re_trans = 5/times10^5.
● Flow around immersed bodies: Here again the development of the boundary layer is
what determines the type of flow that exists. So the length in the direction of the flow
is the appropriate value to use. E.g. for wings Re is based on the chord length. Use
Re_trans = 5\times10^5
^
Did people use A = π*R^2 or A = 4*π*R^2? Using the area of a sphere I got 0.843m - look ^ I
thought it was the equivalent area if you looked at the sphere face on. As in the area the
sphere looks like when looking from the direction of the wind velocity? ie: a circle
ii.
T=sinθ/Fd
T=√(Fd^2+mg^2)=√(250^2+98.1^2)=268.56N (agreex10)

sumFx = 0 = Fd- Tsin (theta) -> T = Fd/sin(theta)


sum Fy = 0 = Tcos(theta) - W . Sub in T = Fd/sin (theta)
-> Fd/tan(theta) = W -> theta = atan(250/(10*9.81)) = 14.3 - (Theta shouldn’t be 14.3 it should
be 68.57)
-> T = 250/sin(14.3) = 1012N

Or, even easier, Ty has to equal weight and Tx has to equal drag. Use good ol’ Pythagorean
Theorem --> 268.56 N (Agree x1, no idea where red has got 14.3 degrees)

iii.
θ proportional to drag, which increases linearly (u^2 term in drag, isn’t linear) until flow
begins to transition to turbulent, causing a slight dip in the curve until flow is fully
turbulent, then continuing to linearly increase as speed increases.

Initially the flow around the sphere is Laminar. During this phase the angle will increase
steadily as the forward speed increase. However, once the forward speed is sufficiently
high, so that transition from laminar to turbulent occurs, the drag acting on the sphere
reduces. Consequently, the angle will decrease as the Reynolds no transitions from laminar
to turbulent. Thereafter, the angle will increase again. (Its in week 11 tutorial)

This is what I found:

The vertical axis corresponds to the angle (in degrees), whereas the horizontal one refers to
the velocity of the helicopter (in metres per secon8y8d); the highlighted point represents
roughly the maximum limit we were asked to analyse (100 km/h). Given we are supposed to
use item i’s diametre, the speed at which the flow turns from laminar to turbulent is ≈ 3.4 m/s: very
small, so the above function just uses the turbulent drag coefficient for all velocity range. If we were to
consider this brief interval when the flow was still laminar, this would only change a bit the very
beginning of the curve...
Question 10
I’ll ask a query here if that’s ok :) So we need to calculate frictional losses foh one valve
being closed and the other open, do we still r fluid going down the closed path and back up
(and minor losses from bends etc going down that path) or can we completely disregard the
closed valve path as a path not taken? Seems a bit funny to use a minor loss coefficient for
the T valve if we’re only going to consider the flow choosing one direction from that. Agree:
I had the same doubt… not sure whether I should use - for this T piece - the original T
branch loss coefficient or the 90° elbow one...
Disregard it. It’s kind of similar to current flowing in a circuit; if it’s open, no current will
flow through. Since the middle of pipe 2 is blocked off, some water will flow down the pipe,
but it’ll stop as soon as it hits the closed valve, so the flow velocity is zero. Not sure which
K value to use, though.

Using the Energy equation, P2 is atm, what is P1?atm..


ε/D=0.25mm (it gives you roughness height which is ε you have to divide by D to get
roughness ratio so it will be ε/D1 = 0.001666 and ε/D2 = 0.00125)
Is V1=0??

I got: V = sqrt( 38g/ (2.42 + f66.67) Agree +2 ?? what about the elbow of pipe before the
second valve? I assumed that because no flow is going through the second pipe, the t
junction is just a 90 degree elbow.
ii.
Assume Laminar to avoid re-arranging Haaland?
f=64/(UL/v) => U=fL/(64v)=10/64/1.5E-5f=10417*f m/s
Big pipe, probably going to be Turb. The transitional velocity - in which the flow would stop
being laminar - is 15.41 mm/s in this case (very slow), so it seems more reasonable to start
assuming it is indeed turbulent and iterate from Haaland’s formula.

i - V = √(2g(z0 - z1 + hp)/(f*L/D + Σ(K))) ---Shouldnt it be this ---> V = √(2g(z0 - z1 + hp)/


(1+f*L/D + Σ(K))) Agree with pink (x5) what about the fact that v_0 = v_1? that should get rid
of the 1 shouldn’t it? - they are not equal v_0=0, v_1 not = 0.
Shouldn’t pressure head at the beginning of the pipe = hydrostatic pressure at depth z0? So
that would make it 2*z0 in the numerator

2015:
I think z0 = 0, as the datum they measure z0/1/2 off is the pump inlet it seems. Taking point
0 as the inlet gives z0 = 0. P0 = roh*g*z0 as this is the hydrostatic pressure due to the water
weight above it. V0 = 0 as there is no flow rate due to taking point 0 as just outside the
pump inlet (infinite area = 0 velocity). This gives:
V1 = SQRT( (2g(P0/roh*g - z1 + Hp)) / (1 + sumK + fL/D))
I think it’s a reasonable approach, what do you reckon? Take point 1 as the surface of the
reseirvoir as the question states no inlet losses and they have a measurement to the top

ii - V = 13.3 m/s, Q = 0.235 m^3/s


(Sean) I interpreted their stupidly set out Z values. Zo is 4m, but it’s drawn as below the
surface (I’m assuming sea level is their reference plane otherwise why not just have Zo as
0m?).
i) V = √(2g(z0 - z1 + hp)/(f*L/D + Σ(K))) is the correct equation, but Zo needs to be -4, so
comes out as V=√([22g]/[66.67*f+1.42]) lol
^where is hp? Is it just 10m? hp=pump head (20m given)
Does someone mind explaining the simplification of the terms in the above equation.. What
happens to the V2, V1, P1 and P2 terms why are they cancelled out… ← pump head=
p1-p2/(ro*g) , v1 and v2 are the same, What??? if Pump head = (p1-p2)/rho*g, then why is hp
still in the final equation? <yeah wtf is happening?
Question - since there is no flow going into the other pipe, would we consider the t-juncion
to be effectively a 90° bend - and the K factor reflect that ? (+1 by OP)

I think it should work out to be:


V = sqrt( 38g/(1+f(10/0.15)+2.42)) where sum(Km) = 2.42 (how to get this sum k value??)
shouldn’t we get km = 0.05 for valve 1? since valve 2 is closed. my sum (Km) = 1.37.
is and the pump it 1.42 or 2.42?? can’t seem to get 2.42 if you assume sudden
expansion joint as a T-piece…. sudden expansion shouldn’t be an issue. only matters when
you have sudden expansion into a reservoir, which is not the case.
Why did you get 38g, not 22g? t the 38 comes from doing (z0-z1 +hp) but i thought z0 was
-4? the pump is under water on the floor, if so then you get 22g, thoughts? it makes more
sense that way, if it is -4 then it means that z1 is 5m above surface, honestly never seen an
outlet of a fountain pipe protruding 5m above water surface.
Except the ones in the UQ lakes...
Could you please elaborate more on how you got 2.42?
sum(Km) = 2.42 if the t junction is assumed to still have a loss coefficient of 0.64;
2.42 = 2*(0.34) + 0.64 + 0.1 + 1
the one comes from factoring out the v^2/2g term and inputting it in with the hf and hm
terms

ii) Iteration… :(.... f=0.0225 V=8.6m/s Q=0.152m^3/s - I got this - I got this as well - So did I
Does this answer use the T-junction K value or a 90 degree bend, seeing as it doesn’t really
flow both ways, also does it include a kinetic energy term at the exit, as well as a sudden
expansion K value
Where is point 0 and point 1 set in this solution? If you use bernoulli between pump and
exit, there is a difference in the pressure, which isn’t included in your solution. And if you
use bernoulli between surface and exit, you have different velocity and also the difference
in height should be 1, and not 9.

Anyone get V=9.75m/s, Q=0.1723m^3/s, +3 make sure that in the initial energy equation, v1
and v2 cancel. They dont cancel you dingbat

b)
Same business as above with the heights.. i - Dout = 0.164 m^2
wouldnt v1 change because of the nozzle… and if you are calculating V1 just before the
nozzle then (P1-P2)not=0
D2 4 ((z 2−z 1−hp)∗2 g−V 22)
D1
=
√(1−fL /D 1−Σ K )∗V 22
=0.9312

^what is this magic ^yeah how? I’ve been reformatting everything then you show me
that this is possible.. maybe don’t say otherwise I won’t study tomorrow, too busy
equationing. Did we ever work out how this magic was done? >INSERT >EQUATION Not
that hard to work out. GET REKT
Dout=0.1414m, Ratio=1.414, Q=0.2355m^3/s I got this. me too. Also I got this << ratio is
0.707
I understand to do the energy analysis on pipe 2, but which part of your equation bring the
nozzle into it if there is no frictional losses?1
yeah?!
i) use V=√([20g]/[100*f+1.42]) > shouldnt it be V=√([20g]/[100*f+1.42+1]? otherwise you’re
excluding the velocity at point 2 (this gave me D = 0.132m and Q = .208m^3/s), then
Qin=Qout for nozzle, solve for D2.
why is it 20g and not 18g, why z2-z0 and not z0-z2….
ii) 0.236 m^3/s (Q=VA)

c) Any idea how to do this??


Does the head split between the two pipes evenly? 10 each way? if so from looking at the
energy balance does that mean if head is halved then V^2 is halved?
For a 2mark question I’d say that’s a reasonable stab. I’ve got no other. V’s wouldn’t be
halved, because they’re depended sprt(Hpump) so they’d be 0.707*Vold
its Δz + head of the pump so this wouldn't be that simple; 2g(z0 - z1 + hp) ^
you know how much head is supplied, thus the head in each pump must equal that. then
solve for the velocities from that as q will be constant throughout
Won’t the velocities be the same? Along each streamline, there is still the same head-loss
and the question specifies that the pump head increase is 20m regardless of Q. Q out of the
pump will increase but the two pipes should behave the same as individually. ←- Good point
because we would be increasing flowrate through pump which in the real world would drop the head
produced, but because we have a constant Hp, the velocities won't change.
Question 11
a)

To help us understand:

00.
This reference frame stuff is just too confusing (especially due to the notation they use)...

I think the secret is in one of the equations provided in the formula sheet:

Vn1 and Vn2 are respectively the velocities before and after the flow crosses the shock in its
reference frame (i.e. when we take the shock as being stationary). Then, we are left with:
M n1 √❑

One equation, one unknown, hence solvable (inclusively by hand, although being cubic). It
is not necessary to iterate in this method to reach the final answer of Mn1 ≈ 4.6.+1 <--is this a
valid solution because its fucking genius. Im just a tad worried its a little too easy (in a
sense that theres not many steps taken for a question worth 8 marks)

T1-150
√(kRT1)=√(1.4*287*150)=245.5

Ms=4.5
Vs=Ms*√(kRT1)=4.5*245.5=1104.75 m/s
should it be: Ms = 900/245.5=3.66? Is this too simple? PLEASE HELP!!
B.2
@Ms=4.5:
Vs/V2=4.8119
V2=1104.75/4.8119=229.59 m/s
T2/T1=4.8751
T2=4.8751*150=731.265
V2+Vs=229.59+1104.75=1334.34 m/s
M2=229.59/√(1.4*287*731.265)=0.4236

@M2=0.4236+
Ms=4.5 %fuck. back to where I started

Goal: V2-Vs=900

a) M_s = 4.616
I got Ms = 4.09075 ?

V_S - V_2 = 900

M_S*√(kR*T1) - M_2*√(kR*T2) = 900

T1 is the stagnation temperature; to simplify the prior line,


T1/T2 = 1 + 0.2* (M_2)^2 (this relationship doesn’t hold over a shock wave. This is used for
calculating the stagnation temperature of an isentropic flow, shocks generate entropy)
T2 = T1 / (1 + 0.2* (M_2)^2)

therefore,

M_S*√(kR*T1) - M_2*√(kR*T1 / (1 + 0.2* (M_2)^2)) = 900

Iterate and find that M_S = 4.09075

V2 and Vs are in the lab reference frame. In order to use the tables/equations you need to
convert to shock frame (I used Va and Vb to represent the flow velocities across the shock,
although in the tables they’re V1 and V2). It’s possible to equate the velocity ratios for both
ref frames (i.e. Va/Vb = Vs/(Vs-V2)) then rearrange so that V2=900 is on one side. Since both
Va/Vb and Vs are both dependent on Ms, you can vary the value of Ms via trial and error to
make V2 equal to 900. Also, I would recommend using the equation for Va/Vb rather than
the tables, because that’s a lot of interpolation.Where did you get that equation from, we’re
stuck on how to find Vs? Vs = √(kRT)*Ms. The equation above is derived from the reference
frame conversion. I had to look it up online because I couldn’t find anything useful in the
notes.Ahh, that makes sense. From the notes, they have V1 (or Va in your case) to be 0?
Wouldnt that cause that equation to be 0? So Va and Vb are in the shock reference frame,
which means the shock is seen as stationary. From our point of view, for Va to equal 0, it
would need to be travelling the same speed as the shock, which is isn’t. The only 0 velocity
in this question is the velocity on the other side of the shock from V2 (in the lab frame, so I
suppose it would be V1). How is V1 found then? As I see it at the moment V1 and V2 are
both unknown as you are trying to get V2 to 900? V1 is 0, V2 is 900, you are trying to find
Vs. The V1,V2,Va,Vb notation is very moo and it was annoying of whoever wrote the test to
use V2 there. Yeah it has made it so confusing.. Cause I figured by that logic V1 = Va and V2
= Vb, then when thats applied to the equation you listed above for Va/Vb= Vs/(Vs-V2)
means that you can’t really solve it… I’m very lost… haha. I know how you feel, I’ve been
stuck on this for ages. Va = Vs -V1 and Vb = Vs-V2. Apparently that’s just the way these ref
frame transforms work. Yeah haha, putting it into that form makes more sense than before
so thank you!!
so Ms=4.5 and Ts=150 we get Vs=1104.75
then using table B2 we get Vs/V2=4.812
V2=229.58, but reference to lab, Vs - V2 = 875.17
then we adjust Ms so V2 lab ref equal to 900
is that right?????

For me, I used so Ms=4.5 and Ts=150 we get Vs=1104.75


then using table B2 we get Vs/V2=4.812
V2=229.58, but reference to lab, Vs - V2 = 875.17
then we adjust Ms so V2 lab ref equal to 900
Then 900-875.17 = 24.83; in which I add this value to the initial Vs, 1104.75
It will be 1129.58m/s; and I will get my Mach number of 4.6 with formula M = V/A I got similar
to this x5

Can someone explain the difference between the M_s mentioned in the question (900m/s)
and the M_s you guess and iterate in part a) (starting M_s = 4.5)? V2 is given as 900 m/s, not
M_s.

I did this:
Vs - V2 = V2_lab → Ms*√(kRT1) - M2*√(kRT2) = 900 → Ms = (900 + M2*√(kRT2))/√(kRT1)
Find M2 and T2/T1 from table B.2
Iterate until you get Ms with 2 significant figures (Ms=4.616). Quite easy...

this question is sick!o x1 agree

b)
T2 = 762.19, M2 = 1.626, p1 = 4050 kPa (Confirm same results - AGREE x4)
How did u get this T2? I looked up the B2 for M = 4.6 T2/T1 = 5.0523 → T2 = 757.845 is this
correct??? and how did u calculate p1? ????
How did you get the mach number M2? ????

Please can someone explain how to get M2?


M2=900/sqrt(1.4*757.8*287)

I got my T2 = 757.845K, P1 = 4.07kPa using M2 as 1.6 x5

Can anyone confirm which is right? For T2 - 762 or 757???


^ Im pretty sure its just because in part A before you were meant to find Ms to be 4.616…
Instead of people using just 4.6. The first answer uses Ms = 4.616
The question says 2 significant figures, which is 4.6. 4.616 is 4 sig figs?
^ confirmed

c) β = 64, Mn2 = 1.461, T3 = 987 K, H2 would ignite (how do you find β, when Ma1 = 4.616?)
ma1 is the 1.626 found in b, howd you get 64 though? is there an equation you can use? I
just used the graph and got 65 how do you get Mn2, Ma2 from the graph is ~1
How did you find T3? Also, I found using the graph Ma2~1.1 .B=64 Then Man2=Ma2*sin(64-
15)=0.83.

This doesn’t make any sense. M1 is the speed before it hits the oblique shocks so
obviously it cannot be identical to the speed after. what we calculated in part b was M2
which is the speed after the shocks. this is not equal to M1 which is the speed before. Does
anyone actually know how to solve this part?????

Are we just assuming that the flow in front of the oblique shocks is at M2 as calculated
before, and the slower flow after the oblique shocks kinda melds into the M2 flow before the
normal shock, so we can just ignore it? That’s kind of a weird assumption

I’ve got Mn2 as 1.44 using Figure B.1 since theta is given as 15 degrees. I’m not too sure
whether it is right or not. My T3 is 553K by getting the stagnation temperature and then
using the Mach number to find T3 from table B1.

How do you get 1.44 from theta =15? which other value are you using?

Using theta - 15, we can obtain the beta. Use the theta value and head down towards 1.63
(approximation).. You can obtain the beta which is around 60-65 degrees. From there,
Ma1sinbeta.

so 1.63 is your m1? - Yeap yeap…

Yea but that doesnt make any sense because what 1.63 is our V2 from before which is the
speed after it hits the shock not before. the speed before the shock- m1 - will be different
no?

Just saw the graph and you could be right, might have to ask the lecturer about this.

Right, like i understand if m1 = 1.63 how you get that but m1 doesnt = 1.63, 1.63 is the speed
after the shock not before which would be related to m2 but we have to use trig to get m2
and then i think we could solve the rest of the part.

This is what i did, m1=1.63 because this is the mach number relative to the lab, remember
that the air in front of the shock isn’t moving at all relative to the lab, so you couldn’t even
approach the question that way. From this and figure B.1 i found at theta =15 and m1 =1.63
beta=64 degrees. I used The normal shock equation (remembering you just use Mn instead
of M) to get T2/T1 = 1.39 then remembering that the temperature after the shock is 757 we
can find T2 = 1051K so yes the fuel would burn. Jet fuel does melt steel beams.

d) Can’t draw conclusion as T2 = 762 K too hot to resemble atmospheric conditions.


Question 12
Assume isentropic flow for 1-2 and 3-e
Stagnation temp and pressure constant until heat addition, where ΔTo=Hin/Cp
Choked exit, Me=1
A2=A1*0.2=0.02*0.2=4E-3

a)
M1=20.8/√(1.4*287*298)=0.0601
To1=298(1+0.2*0.0601^2)=298.215 K (agreed x5)

m.=ρU1A1=1.2*20.8*0.02=0.4992 kg/s (you assumed density of air is 1.2? Use P= rhoRT)


but doesnt that give rho = 0.0012?
b)

B.1:
@M1=0.0601:
you are a VERY smartass
A*/A1 = 0.0601/0.1*(1/5.8218)=0.1032 Are we meant to have memorized this formula?
A*=0.1032*0.02=2.0647E-3
What is this ^ formula!?

@M2: A2/A* = 4E-3/-.0647E-3=1.9373


P2/Po=0.9564 from table b.1
P2=Po*0.9564=96.83kpa approx
whoever did this step, I think you used the wrong interpolation values - these values look
like they are for the Mach numbers 2.1 and 2.2 I Agree

Well I got P2 = 97.9 kPa and M2 = 0.322 Got this as well x7


You also get this ^ using the formula on the updated formula sheet for this year 2015

I got P2= 94.45 and M2=0.322 (i got the same as this) Me too! (agree x6)I get effectively the
same (How are we meant to judge that we need the subsonic branch of table B1 rather than
the supersonic one?) I assume you say its subsonic throughout the pipe thing, because we
are told that it is choked at the end. Narrowing area will increase mach number until the
flow is choked, then it’ll chill at one cause a converging nozzle causes supersonic flow to
go to mach1. That’s why they use converging-diverging nozzles, so you can choke it and
then accelerate the mach 1 flow past sonic, into supersonic.

How to decide which part of table B.1 to use:


You know A1, and A2/A1, so you can calculate A2. Then you can find A1/A* and by that find A*.
Then you’ll see that A* < A2 → you have not reached choking yet, and the flow must still be subsonic
→ use subsonic part of table B.1

M2=0.3+(1.9373-2.0351)/(1.5901-2.0351)(0.1)=0.3220

How is this done? I’ve got my P2: 77.6kPa and M2: 0.672

Why is Bernoulli not applicable here? Isnt this a simple nozzle?


I think it’s got to do with it being a compressible flow not incompressible flow. lollllll
I’m REALLY confused with this. How exactly are we supposed to work this out? Especially
since M1 is so small. Have a look at what lad/lass said below, can’t interpolate because of
that infinity, so use the A/A* equation (hopefully it will be give to us).

I just bernouli’d it since the mach speed is so low, so its not really compressible. giving m2
= 0.301

I worked out the A/A* using formulas that worked well :) x2


2015 Class: Can we assume conservation of mass can be applied and maintain mass flow
rate and density seeing as compressible effects only come into effect when M>0.3?
Doing this I get M = 0.3005.
yes i also got this by this method^
c)
To3=To1+Qheat/Cp/mflow=298.215+100/1.004/0.4992=497.32
@M1=0.0601: ← why are you using M1? Why not M2 as the flow enters the constant area duct?
NVM, read further down...
To1/T*o = 0.0601/0.1*0.0560 = 0.033656
To* = 298.215/0.033656=8861.75 K (To3 < To* so flow won’t be choked)
That value is for T/T* and not To/To*. I think the correct value to use is 0.0468
Thus, To* now becomes 10602.70
^Not sure where you’re getting your To 1/To* value from, but I’m getting 0.389 - worked out
using the To/To* equation. Then, To* = 766.54 K +1. - I get this too, me tooAgreed
Should To* be found using To1 or To2- as I believe it was assumed that To1=To2
There is no enthalpy change. so To1 = To2. Hence it does not matter which one you use.
The stagnation temperatures at 1 and 2 are the same. But you should use M2, right?
Yes, but since To1 and To2 have different Mach numbers, then the ratio To/To* will be
different (from the Tables), thus if the same value for To1 and To2 is used, two different To*
values could be found using these To1 and To2 values
From what I believe, the T1 and T2 are different and that would affect the results. Wait, I see
what you mean. Yes you are right. It has to be evaluated at Mach2 (which is 0.322)
In the solutions for week 10 in class problems, they use To2 @ Mach2 (which is equal to
To1) to find To*. Cool :D, glad to know I’m not insane(LOL)
@To3/T*o=497.74/8861.75=0.0562
M3 = 0.1+(0.0562-0.0468)/(0.1736-0.0468)*(0.2-0.1) = 0.1074

The stagnation pressure Po3 will have also increased from Po2=101.256 kPa with the 2-3
heat addition but the static pressure P3=P2 (static pressure changes with heat addition,
should be using Po/Po* relations)
P3=10.116kPa
Redone (evaluating at T02 as discussed above):
@Ma2=0.322
To2/To*=0.38696 (Table b.4) (using eq = 0.389)
To*=298.36/0.38696=771.03 (=766.54)
P2/P*=2.0938 (Table b.4)
P*=94.45/2.0938=45.107
@Ma3: Check this interpolation. I think it is meant to be between M = 0.4 and M =x 0.5- yeah,
I think o right/makes more sense, I just saw the value at ~3.1 and grabbed that. Fixed.
To3/To*=497.88/771.03=0.6457
Table B.4: Ma3=0.4718 (Agreed x7)
P3/P*= 1.829 (Table b.4)
P3=45.107*1.829=82.5kPa
Agree with revised purple (M=.47, To3= 497.7K, P3=83Kpa), shouldnt this say M=.47?,
elsewise, i agree. Yeah, can’t be supersonic yet as flow hasn’t been choked

d)
(Sean) M3=0.47 as above
A2=A3=0.004m^2
Tableb.1 A/A*=1.415
A*= 0.00283m^2 (agree x2)
Assumin x3 - this is Pexit)
T/To=0.833; T= 414k Te=497.7*0.83333=414k ? yeah, calc error. My bad.
g Isentropic relations To3=Toe=497.7k and Po3=Poe=94 Kpa
Flow is choked, so use Ma=1, P/Po=0.5283; P= 49.7 KPa (agreeWIth T Find speed of sound,
which is Ve= 407.8m/s (agree x3)
F= 193.1 - 145.3= 47.8N. Not exactly significant but then he’s trying to recycle waste energy
so hardly surprising. Almost the same but F= 44.7N by finding new mdot based on row =
Pe/R*Te
Why do we need to find the new temperature for new speed of sound? Presume this is only
in cases with heat addition?
Speed of sound changes whenever the temperature changes, and the temperature definitely
changes here

B.1:
@M3=0.1074:
A3/A*=5.8218+(0.1074-0.1)/0.1*(2.9635-5.8218)=5.6103
A*=4E-3/5.6103=7.130E-4 (Ae=A* will choke the flow)

P/Po=0.9930+(0.1074-0.1)/0.1*(0.9725-0.9930)=0.9915
Po3=P3/0.9915=10.116/0.9915=10.203 kPa %should have increased if above
statement is correct

@Ae/A*=1:
Me=1
Pe/Poe=0.5283
Isentropic so Poe=Po3=10.203kPa
Pe=0.5283*10.203=5.3902kPa

Ve=Q/Ae=0.4992/7.130E-4=700m/s but the density won’t be const., need to use rho=P/RT


F=Q(Ve-V1)+Ae(Pe-P1)=0.4992(700-20.8)+7.130E-4 + (5390-101000)=-95.27kN

e) no sir. Care to elaborate? I’m not quite sure what they’re asking…

Possibly something to do with Pe being less than Patm so it’d create a vaccum sucking air
into the thruster?

Not positive about this, since it’s only a converging nozzle, but for a converging-diverging
nozzle, it would create a series of oblique shocks at the exit to bring Pe up to Patm
Question 13
Axial Turbine? NOPE YEP ahhh no

Part A -Taking a stab- let me know if i screw something up


[INSERT PRETTY PICTURE HERE] Could you upload/link to the velocity diagram? Very
confusing. Thanks

(Could anyone plz tell whether the above method in pic right or wrong)
Also is there a better quality image? The subscripts aren’t clear
Part B - shock free so trying to “catch” the w1 air off the stator blade onto the rotor- match
the angles
u= ((3000*2π)/60)*0.35 =109.9557 m/s
Va=Q/A =20/0.15 =133.33 m/s (Can anyone plz tell why axial velocity equals to Q/A here?)
Vt1=Va / tan45
Vt1=Va (because 45°)=133.333 Vertical component of w1 = Vt1-u
so B1=tan^-1((Vt1-u1)/Va) - Via the Z rule from year 11 trig
B1=9.9445° - i hate trig I got this as well x 5

Part C - Swirl free so Vt2=0


V2=Va=133.33m/s
This means that w2+u=V2 to give purely axial flow in the absolute frame
tanB2=u/V2 = u/Va
so B2=39.51° - quite steep because you have to shoot all your relative velocity up to cancel
the u value vertically Agree with this x4 Anyone else get 49.608°?

Part D Use the geometries you found to determine torque and power
T=ρ*Q*(r2Vt2-r1vt1)
vt2=0, r1=0.35, Vt1=133.33, Q=20, ρ=1.2 (not specified but look at all its other properties, if
its not air there is something seriously wrong) Agree with this sorry forgot ->Possible
difference here, we are given T = 1200K, and P = 20MPa, so by ideal gas law can we say P = ρ*R*T, →
ρ = 20e6/(287*1200) = 58kg/m3? I agree x5
Sub that all into the torque formula
Torque comes out at T= -1119.972 N-m - Extracting work- so this works out << i got -54200
Nm (x5)

Power(via the other formula)


P=ρ*Q*u*(Vt2-Vt1) = -351.857 kW Extracting so works out
So due to the difference in densities, are people gettting power results of arround
17.04MW?? Yes, I don’t think I dropped any decimals and got 17.025 MW. i got 17.03MW
same agree x4
I think you have to use the given pressure and temperature to get the density otherwise
there is really no point as to why they gave it to us in the first place (also leads to part e),
but makes sense to use this info to calculate density)...also, does having an air density so
high have anything to do with part e) below?

Part E
It is an acceptable assumption to make because a) Ingo said it and b) i read somewhere that
because the pressure rise across a single blade is small it is ok.
Would break down first after the stator blade as this is the point of highest absolute V.

V1= 188.56 m/s (vector addition).


M= V1/ (1200*287*1.4)^½ = 0.27. Compressible effects are negligible when m< 0.3 so all
good. Plus what Mr purple above has said.
2011
Part A
1. D
δV/V=√((0.5*3/670)^2+(-0.5*0.02/1.2)^2+(0.5*0.01/9.81)^2+(0.5*0.002/0.13)^2)
=0.0116
=1.16%

2. B
x=0.25m0
U=25m/s, u=12.1m/s, u/U=0.484
7.1:
y*√(U/ν*x) ~ 1.5
y=1.5/√(25/1.5E-5/0.25)=0.5809mm

3. C
m*g=9.81*1000*4/3*π*(20E-6)^3=3.287E-10
At terminal velocity:
m*g=Fd=Cd*0.5*ρ*U^2*A ahh… I was using Cd from the formula sheet instead of the given
stokes relation =24/(U*40E-6/1.5E-5)*0.5*0.7*U^2*(π*(20E-6)^2)
=3.96E-9*U
U=2.63E-9/3.96E-9=0.664m/s
t=10000/0.0664/3600=4.18 hours
out by a factor of 2?

I simplified the equation down to get this:

(⅓)*(ρ_p / ρ_air)*g*(4/3)* (D/2)^3 / (D*ν) = V <~~What equation did you simplify to get this?
Just equate the Cd=24/Re from the question and Cd=Fd/(0.5ρ_air*AV^2), Re=VD/ν, For Fd
you need the mass: m=4/3ρ_particle*π(D/2)^3 (*g=Fd). It kind of just falls together once you
rearrange it a bit. When calculating the Reynolds Number why are you using D?
Because the only dimension that affects the flow around the sphere is its
diameter, external flow and all. So it’s the “Hydraulic Diameter” in this
situation.
If you think about it the length of the fall isn’t affecting the flow, if the ground
were 2 meters closer the Reynolds number, or rather the turbulence that it
represents, wouldn’t be affected, while if the sphere was much smaller it
would have less of an impact on the flow going past it.
A=π(D/2)^2 btw, doesn’t matter that it’s a sphere.
V^2/(DV/ν) = Vν/D = Fd/(12ρ_air A) -> V = Fd*D/(12 ρ_air Aν)=0.08304m/s
V = 0.083 m/s
t = 33.45 hrs I got this as well me to YEAP +3
I got this as well.
Simplified to the force balance to U = ((D^2)/18)*(ρ_p/ρ_a)*(g/ν)

4. A C[1] 3.98m/s
D=0.6m
L=3000m
hf=80m
f=0.016
v=4.43m/s

guess f=0.03 (Atula - he says to start with f=0.016, that will simplify calcs. you still get the
same results obviously, but just saying) With f=0.016 like he says you only need 1 iteration
V=0.56/√(0.03)=3.233m/s
f=(-1.8*log(3.959E-5+3.45E-8/3.233))^-2=0.0159
V=0.56/√(0.0159)=4.441m/s
f=(-1.8*log(3.959E-5+3.45E-8/4.441))^-2=0.015
I get it converging to around 4.17 after the third iteration, kind of between two values
^^ that’s because you’re fucking retarded x61 agree

5. C
θ=30°
M=4
Can someone please add when a detached shock forms?
(where to find more info?)
There are solutions for β, therefore an attached weak or strong shock will form. And if there
isn’t? What does that mean for this question say if there were no solutions for β??? Cheers
If theta>theta_max a detached shock occurs allowing for smooth deflection around the
wedge.
Why is it not Prandtl Meyer expansion fans? ←- u wot m8? x69 agree

6. B) Why does B necessarily happen here? why does a normal shock have to form in the
diverging section? I believe it is correct just don't understand 100% why. ????????
I second the want for explanation someone who knows what they are talking about plz
A) would only be possible if there was supersonic frging part (I thilow in the convenk), This
isn’t english right? which isn’tq possible in this situation (gas in reservoir) Why is a not
possible?
C) This will only happen if the back pressure is less than the exit pressure (again, I think).
I guess that it still needs to be sonic at the throat and the normal shock in div. would slow it
to the subsonic flow as per question
Y’all are fucked it’s D)
7. B
V1=4*√(1.4*287*298)=1384m/s
B.2:
@Man1=4:
V1/V2=4.5714
V2=1384/4.5714=302.7m/s

V2lab=V2+V1
I thought that the lab reference frame was V1-V2=1088.something≈1100m/s ∴B
will have to agree with you there agreed, the shockwave is moving away from the air behind
it at 302.7m/s but is moving towards the untouched air (lab reference frame) at 1384m/s thus
the air behind it is moving at 1384-302.7=1081.34 m/s in the laboratory reference frame.
Make sense if u draw the directions of V1 and V2, they both pointing to the left
Does anyone have a diagram/drawing of which way all speeds point (to help with reference
frames?)

8. n/a Pretty sure C is one of them anyway

GET ABSOLTUELY FUCKED


Part B
Question 9
(Sean)
a)Fluid continues its run or unnecessarily retarded phrasing. What I assume they mean by
“all flying conditions” is, is it turbulent at minimum flight speed?
Can be considered turbulent if L_lam<0.1L ← is that in the equation sheet? or from the lecture
slides? ie if it’s only laminar for 3m we’re all good
kinematic viscosity= ν = 1.5*10^-5
Re(transition)= 5*10^5 = V*L_trans/ν; There’s no such thing as veta mate, it’s nu (ν)
Ltrans= 0.267m which is much smaller than 10% of L, therefore can model as turbulent for
the whole length

b) trailing edge: x= 30m. V=130/3.6= 36.11m/s


Turbulent table equation for δ 0.362m (seems large..)I got this. me too. also here and same
here +1

c) τ(one edge)= [½*ρ*V^2*Cf] Where does the 2 out the front of that come from?
where Cf= 0.027/Rex^(1/7)
τ(one edge) = 1.5925 Pa yep agree with this as well
Total τ(both edge) = 3.185 Pa
I think you are confusing tau and the frictional force. tau should be in pascals, and i dont
think you need to take account for both sides, ie that two in front of the brackets in your
first line of working shouldn’t be there, (i think)
I remember the tutor saying that you have to take into account both sides.
You’re correct it should be in pa (I had it written in my book as Pa but goofed). As to
doubling it you definitely need to take into account both sides if both sides have a flow past
them. Friction force is integral of Tau over an area so if you have to take number of surfaces
into account for one then you have to do it for both.
Much confusion over if they want shear stress of 1 side or shear stress of both. In the end, I
dont think it really matter. Everyone is in agreement that you need to take into account all
“submerged” surfaces when calculating drag force, which is likely to be a far more
important and weighty question.
So shouldnt the 2 in the first line only apply when considering total? not one edge as is
considered in the first line?

d) So, resolved the tension in the cable to vertical forc m


e (mass*g) and horizontal force (drag)

Fd= int (τ_w dA)= int(0,30) [2*½*ρ*V^2*0.027/Rex^1/7)*h dx]


ρ=1.2 V=36.11 h=3m ν=1.5*10^-5 integrate Rex from 0Fd= 334.44 N
-30

tan(θ)= o/a= Fm/Fd = 90g/334.44 You didn’t take forces acting at centre of banner?
θ = 69.25° I got this too. Yup x5
i got 111°…. so the banner is in front of the plane hahaha
^ I bet there is a mistake in your trig, 180 - 111 = 69 degrees.
If you take into account that the forces act at the centre of the rigid banner, theta = 111.8°
Makes sense because mg is acting 15m from corner, whereas drag is acting 1.5m from corner
Question 10
a) T0 = T(1+((k-1)/k)M^2)=220(1+0.2*4)=396K +2
b) From Figure B1, with θ=15°: M β=a1=1.5,45° ? how to find beta from the graph? -
Calculation from trig actually gives 1.59 it says
using figure b1 though (plus says 2 s.f. so blue person should be 1.6)
c) After the Normal shock: Ma2 = 0.7011 (Table B.2: Normal Shock Relations)This is the
normal Mach number, i think we need to calculate Ma2 = Mn2/(sin(beta-theta). This Ma2
should be used to calculate A2/A* for calculation in part d. That’s what I think.
I understood it that for a normal shock we have to use the mach number perpendicular to
the shock so I used Mn1 = Ma1*cos(theta) then the tables. Can anyone else confirm this?
Mn,2 = Ma,2 sin(beta), then table! straight from lecture example Is this meant to be beta or
theta???? Theta makes a lot more sense from the diagrams. ???????????

d) A3/A2=3, A3/A*=A3/A2 * A2/A*

without interpolating (using Ma2=0.7 rather than 0.7011): A2/A*=1.0944 (Table B1)
->A3/A*=1.0944*3=3.2832 -> Ma3≈0.2 (again no interpolation)
A/A* of M=0.2 is 2.9635
A3/A*=3.2832, interpolate between M=0.2 and M=0.1 gives M3=0.189
e) q = 2 MJ/kg, cp = 1004.5 J/(kg.K)
Ma3=0.2: T03/T0* = 0.1736 (as T03 = T0 from start of question) -> T0*=2281.1K
Wouldn’t T03 != T0 since the stagnation temperature would change over the shock?
http://en.wikipedia.org/wiki/Stagnation_temperature the upshot of this is that the total
energy doesn’t change over a normal shock, so total enthalpy stays the same. Both velocity
and temperature will change, but they change proportionally and the stagnation
temperature will remain the same. This is because we assume the shocks are adiabatic.
http://en.wikipedia.org/wiki/Shock_wave#Normal_shocks (end of first paragraph)
The oblique shocks are assumed to have equivalent
normal shocks, which is the point of the prior questions and the figures at the end of the
formula sheet, then they too are treated as adiabatic.
Pretty much no heat change = no change in stagnation temperature, T0.
T04 = T0 + 2*10^6/1004.5 = 2387.04 -> T04/T0* = 1.046 ≈ 1 ∴Ma4 = 1
To1=To3=396k (Why is To1 = To3)? Because none of the processes before heat addition will
actually affect the stagnation temperature, shocks only have an affect on the actual
temperature, as does choking.Well put.
For m3=0.189 To3/To*=0.1584
To*=2500k
q=2*10^6= cp(To4-To3)
To4= 2387k
To4/To*= 0.9547, corresponds to M4=0.7834 (Thelre is two To/To* = 0.9547 values on table
B.4 why did you pick this one? Is it because the flow is already subsonic (choked) so you
choose that value? ) subsonic != choked, sonic (Mach 1) = choked
yeah, it starts off as subsonic (0.189) and adding heat will cause it to go towards sonic, but
can’t go supersonic. Once it becomes choked, does continually adding heat cause M to
decrease or does it just remain at 1?
f) No. Choking the flow is reaching the sonic condition (ie reaching a mach number of 1). If
the area had not increased the heat addition would have occurred from Ma2=0.7
This means that T02/T0*=0.9085 -> T0*=435.88K
The T04 would remain the same (same stagnation temperature initially and same heat
addition). This means that the T04/T0*=5.4763.
That’s not possible, the T0/T0* values go below 1 again after mach 1 on the complete tables
(not given in the 2011 exam, they only go up to mach 1).
The number being above 1 means that the flow is choked and more heat is put into the
system than is required to make the combustor reach the sonic condition. In either case,
supersonic or subsonic, adding more heat will just make the system go closer to sonic.
Making the answer to
the question no, as the flow is well and truly choked.

No it wouldn’t. Extra area dropped your mach number from 0.7 to 0.189. If it stayed at 0.7,
this reduces your To* to 435.88k. You’d choke the hell out of the flow. Increasing area of the
flow dramatically increases To*, allowing extra heat addition.

(So what is the overriding consensus??) … We both said the same thing (no), I just said
more about it. Yeah, answer is No, we both said that. GIMMMMMMPPPPPPPSSSSSS
Question 11
Very similar to Online Problems- Q2
a) See diagram from online problem 2
b) For no swirl-> Vt1=0, therefore Vr1=V1
Q=V1*A=V1*(2*π*r1*b)
V1=1.5/(2*π*0.35*0.08)=8.526m/s

u1=w1*cos(β)
V1=V1=w1*sin(β)
u1=V1/tan(β)
u1=8.526/tan(25)=18.28
u1=ω*r1-> 18.28=ω*0.35
ω=52.24rad/s
ω=498.8RPM (approx 500RPM) I get 497.33, error is rounding I’d assume

c) Use continuity to find vr2 Q=VA


1.5=Vr2*(2*π*r2*b)->Vr2=4.59m/s
w2cos(β)=u-Vt2
Vt2=u-Vr/sin(β)=52.24*0.65-4.59/tan(15) agreed

`^^^ If you

look at the vector diagrams w2cosB is in the opposite direction to u2 and VT2, s
o VT2 should be u+Vr2/tan(B) Not too sure about that. My vector diagrams have u2 and vt2
in the same direction.
Yes but VT = U + wcos(B)
w = Vr/sinB
Vt = U2 + [Vr/sin(B)]*cos(B) = U2 + Vr/tan(B)
pls let me know if you do spot an error.
In the worked solutions (for 2-c), they have Vt=u-w*cos(b). I assume it’s like that as you are
dealing with magnitudes (not 100% on this, I’m just going with the worked solutions)
^^ Cheers

Vt2=16.825m/s
P=ρ*Q*(Vt2*u2-Vt1*u1) (Vt1=0 as above)
P=998*1.5*(16.825*52.24*.65)=855.25kW
T=ρ*Q*(Vt2*r2-Vt1*r1)
T=998*1.5*(16.825*0.65-0)=16371.5Nm
H=P/(ρ*g*Q)
H=855250/(998*9.81*1.5)=58.23m agree x4 with above T,H and P

b) From Bernoulli:
hp = hf
=f*(L/D)*(V^2/2g)
Want hp = f(Q) to compare both relationships
hp = f*(L/D)*(1/(2*g*A^2)*Q^2
= 0.1135Q^2 I get this

(Plot the two functions together and) find the simultaneous solution
Q = 2.968m^3/s (I get 2.356 m^3/s x3) i got 2.256m^3/s
hp = 0.630 (But I still end up getting 0.630 as head)still got this :D

P1/(ρ*g) + V1^2/2g + z1= (same shit but with 2) +hf -hp


P1=P2, V1=V2=V, Z1=Z2
hp=hf
1- Q^2/15 = f L/d V^2/2g
1-(AV)^2/15= f L/d V^2/2g
1= (AV)^2/15+ f L/d V^2/2g= 0.045514 V^2
v= 4.687 m/s
Q= 2.365 m^3/s
hp= 0.63

Solving the function simultaneously I get Q=0.4244 m^3/s ; h_p = 0.02044 Thats wrong.
Question 12
(Sean)
a) M2=3
From normal shock tables M3=0.4752
Interpolate in friction tables to find fL*/D=1.3765
f=0.02, D=0.2m so L*=13.765m

b) Not sure about this one.


I think, using f*ΔL/D= fL*/D(1)-fL*/D(2). For M=3 fL*/D = 0.5222
f=0.02 and L=x=1m
fL*/D(1)= 0.1+0.5222=0.6222
Interpolate to find M1= 3.87 Seems ok cause friction would cause it to drop. Not exactly
sure. Open to ideas agree with this method With corrected fl*/D(1) value, M1 = 3.87 from
interpolation. I am not so sure about this. I think the Mach number has to be lower than
M2=3 for M1. Friction causes it to increase in Mach, so going backwards but keeping
everything else the same, we need to get something lower than 3. This is just my thought,
could be making some mistake somewhere. But using that logic, I get M1 = 0.58. However,
they say diverging section is supersonic, so there my theory blows out :P
You’ve got it backwards mate. You’re only going back as far as the end of the nozzle, so it
doesn’t come into it just yet. Friction causes flow to go towards sonic so as you we’re
going back up the tube, we need to travel further away from sonic. The friction in the tube
causes flow to drop from M1 to M2 (from 3.87 to 3) so M1 has to be higher.

simply put, in compressible flow with friction, the velocity will always tend to mach 1 -
going backwards means going further away from mach 1.

c)
Basically just went to table B.1, calculated A/A* for M=3.879 A/A*=9.5445. Sounds like the
correct procedure, using M1 = 3.87 i got Ae/A* = 9.5445
Did anyone actually worked out throat area? I got 0.0035m^2 (didn’t bother interpolating)

d)
Mucho dodgy.
From B.1 For Me=1, P/Po= 0.5283 so Poe=189.3Kpa=Po3 I think this is wrong, I agree with Poe
= 189.3kPa, as the flow is choked here we can say Poe = Po*. Then at point 3 M3 = 0.4752 by
interpolation Po3/Po* = 1.4019 → Po3 = 256.4kPa.
My interpolation for Po3/Po* =1.2464, which seems about right as it’s closer to 0.5. Po3=
213Kpa

What table are you guys interpolating to get P03? if you use B.3 you’re gonna get P03/P0* =
2.2 if you use B1 you’ll get 0.8?

So does Po3 = Po?? There would be a pressure change from state 1-->2 due to the friction
thought wouldnt there??

From here I don’t know what else to do. I think this may be the answer (seems reasonable
range) But I have no idea why. Po is stagnation pressure, and nothing has occurred that
should have changed it so….
You’re doing the right sort of thing, I cbf doing the working myself, but pretty much the
pressure will change over the non-isentropic processes (ie from the friction).
So you just work it backwards to the nozzle entrance like you did. The nozzle is isentropic
so there is no change of stagnation pressure over it, which is one of the properties of
pressure. So the stagnation pressure at the end of the diverging section is equal to the
pressure in the tank (as in the tank it is stagnated or v=0, so P0=P). So your answer is
probably fine.

From exit, Pe=100, Me=1, table => Pe/Peo=0.5283, Peo=189.29kPa


Exit to state 3: M3=0.4752, Poe/Po*=1, Po3/Po*=1.4019, so using ratios and Poe =>
Po3=265.36kPa
State 3 to 2: M2=3 => Po3/Po2=0.3283, so Po2=808.288kPa
State 2 to 1:Po2/Po*=4.2346, Po1/Po*=9.5445, using the ratios => Po1=1821.8kPa
Since just converging/diverging, Po1=Po=1821.8kPa

e) Flow will get choked was L>L*. Move shocks.


It would move the shock wave. Noobz.. Its was a typo, should read move shocks.But good
on you champ.

Po(res) = 1.81 MPa, you have to work from the exit accounting for friction. For 1d? If so i
agree
Question 13
From Mark Kendall’s lecture:
2010
Part A
1. C
u=3.85 m/s
U=5 m/s
y=0.002m
x=?

Table 7.1:
@u/U=3.85/5=0.77
y*√(U/(νx)=2.6
x=U/ν*(y/2.6)^2=5/1.5E-5*(0.002/2.6)^2=0.197m

From Alex Dalton (can’t be bothered writing answers)


2010 exam:
1. C confirmed :) agree
Is someone able to show working please?
u = 3.85 m/s
U = 5 m/s
y = 2mm

u/U = 0.77 which corresponds to ~2.6 (from table 7.1)

y (U/vx)^1/2 = 2.6, solve for x


x = 199 mm

2. D (3)
can someone explain me how?im getting V1=0.0701 and u1=12.556
Pretty sure because there is no swirl in the incoming flow, V1 = V1_r = 22.28 m/s
Yer and u1=12.57m/s giving β=arctan(v1\u1)

3. C (4)
m=100kg
U=7m/s
mg=Fd=Cd*0.5*ρ*U^2*A
A=mg/(Cd*0.5*ρ*U^2)=100*9.8/(1.7*0.5*1.2*7^2)=19.2m^2
A=π/4*D^2 → D=4.9m
Is it odd the choose to neglect the drag of the box? - I dont think so, there is a reason you
put a parachute on it after all

4. C (3)
Yer you’re on the right track. Basically, p0=2mpa and p after exit is 101kpa. THis ratio of
p/p0 is so large that the flow must be chocked. Thus A_throat=A*, use this to find the ratio
of area to A* at outlet giving m=2.2. Then find the temp at this point using T/T0 then find
velocity :)
Ahh sorted it out thanks heaps man!
Can someone please give me a brief run down on how to solve this? The textbook, lecture
notes and example problems are really not helping haha. Thanks :)
>>>for expansion, use the A2/A1 to find the mach number = 2.2
then use your T/To relations, setting To = 300K
T=152K
V=Ma(kRT)^0.5
=544m/s

Can someone please explain why T/To @M=2.2 =0.5081 with To = 300, wouldn’t T =
590.4K??

5. D D (4)
Use angle graphs and find the minimum m1 were the 30° curve falls

6. C I got D C C C C c- course isnt hard enough lets leave out necessary equations
Q = ρ*A*V
ρ = RT/p - mistake here PV=RT so V=RT/P and ρ = P/RT - C is correct
=0.57kg/m^3
Therefore v = 156.44m/s
a = √(kRT) = 633.87m/s
Therefore M1 = 0.24
Now, as flow choked at end fL*_2/d = 0 Therefore, fL*_1/d = f*ΔL/d
From table, when M = 0.24, fL/d = approximately 9
Therefore, ΔL = 22.7m Therefore D
how do you get velocity from kg/s flow rate?
m’=ρAV=0.7 kg/s
need density of exhaust gas
ρ=1.742
Q=0.402m^2/s
V=204.74m/s
then from m=0.323 find fL/D=4.61 → L=11.53m
Density can be found from ideal gas law : P = ρ*R*T
M= 0.32
fl/d =4.1

7. Biomed shit D < 80% sure


8. C
Does the question seem like a trick, very easy considering all the information they give us
(Who the hell writes an exam with only C and D options) - agreed I was wondering about
that as I went through
Part B
Question 9
a) 3.16 metres How you do this? I got 0.1979m :/ add the frictional loss. I get 3.16m
I got 3.3m but 3.16 is close enough What is the correct frictional head loss? I’ve got 5.142m.
Did you assume e/D = 0? I got f = 0.01128 and V = 1.96 m/s => dZ = 2.96 m
I got v = 1.96*10^-3, can someone please confirm?
Can someone please upload their workings for part a? i’ve keep getting a really small
number :( I get 0.197m as well. You shouldn’t need to add friction, it says smooth walled.
Smooth walled is just an assumption which gives you the e/D line to use on the moody
chart. Smooth walled does not mean ignore friction.
I got f = 0.01128, v = 1.9648 m/s and dZ = 2.9597 m - this was what i did:
dZ=hf =f (L/d)(v 2 / 2 g)
v=Q/ A=1.9648 m/ s
ℜ=vD /ν=1.173∗1 06
f =¿ (e/D = 0 since the walls are smooth, also when checking on the moody chart for a
smooth pipe, f is approximately this value)
dZ=0.01128(800 /0.6)(1.964 82 /2(9.81))=2.9597 m

b) 0.088 metres anyway of getting this without a moody diagram ? i can't work out how to
iterate properly
I am guessing you forgot your minor loss coefficients, Hydraulic Diameter=0.17m=Width
^ agree, I’m getting D = 0.1746 m.
^ I also agree
To solve I set up the energy equation
(ΔP / ρ*g) * 2g/V_2^2) = f(L/d) Where did you put the K factors? Shouldn’t that be included in
the energy equation?
and subbed in to get:
4.6511 = f (L/D)
such that D = Lf/4.6511 and D = 0.1935
Substitute D to find Reynolds number and updated f as 0.02636 What equation was used
here to find f from Dh and Re
Solve D = Lf/4.6511 and D = 0.1700
Substitute D to find Reynolds number and updated f as 0.02725
Solve D = Lf/4.6511 and D = 0.1758
Substitute D to find Reynolds number and updated f as 0.02702
Solve D = Lf/4.6511 and D = 0.17429
Substitute D to find Reynolds number and updated f as 0.02708
Solve D = Lf/4.6511 and D = 0.17468
and so on…
Storing variables on your calculator makes the whole process much easier.
Question 10
a) 0.3375 metres confirmed agree How? Reynolds equation, solve for x. Re = 5x10^5

b) 2.4mm (5)

0.95 millimetres I got 2.4mm. I got 2.4mm using Blasius (lam) since leading up to that stage
it’s laminar flow. I agree with 2.4mm. agree with 2.4 mm as well. Me too! Agreed, 2.4mm.
2.4mm same
How do we know whether to use blasius or momentum integral estimate? Blasius is an
exact solution, only use momentum integral estimate for laminar cases if it asks.

c) 0.278 Pascals I get 0.508 Pa. 2.45 Pa (with Turbulent integral estimate), 0.556 (Blasius)
0.556Pa, 0.556Pa (multiply 0.278Pa by 2 due to their being 2 two sides)
I agree with 0.556Pa

I don’t agree with multiplying by two. For force it would be but stress is independent of
total area, and multiplying by two would only be valid for calculating the force when
Stress*area becomes the equation. Not explaining it well but essentially the stress is force
per unit area. Even though we have two sides to the object it only becomes relevant when
considering the force. I say 0.278Pa.

d) 0.765N dont forget to multiply by width and account for both top and bottom surfaces
d) I get 4.033 N considering turbulent flow along the entire length.
d) Fd= 3.59N (have to account to laminar flow up to transition and then turbulent after that).
d) Fd = 3.59N. ^ same reason.
d) Fd=3.59N ^^same rason
d) Fd = 0.766N
D) Although the question leads you to think that the transition to turbulent is important, the
laminar portion of the board is only 13%. Do you think that we can simply assume turbulent
along the entire length? In tutes, less than 10% was considered insignificant. I guess it’s
your call to make, I just took into account laminar and turbulent to be safe.
Isn’t the board 2.5 m long? Transition at 2.4 mm means that laminar flow exists for a lot less
than 10% of the length right? Transition is at 0.3375m
Question 11
- See Platypus Solution below (next page)
a) Pretty picture
Should be expansion on the top surface, compression on the bottom left and expansion on
the bottom right (Supersonic Expansion Top, Weak Shock Bottom Left, Supersonic
Expansion Bottom Right) (how do you figure this out? Any relevant lecture slides to read
through?) BUMP

b) 18 kiloPascals, 36 kiloPascals, around 9-10 kiloPascals


Top Surface Pressure (Supersonic Expansion)=4.6kPa
Bottom Left Surface (Oblique Shock) Pressure=34kPa
Bottom Right Surface Pressure (Supersonic Expansion)= 6.3kPa
Using the Areas and Force Vectors along each surface the total vertical lift is 13kN
^^ Please explain how you did this!! ^^ ??????????????? WTF IS THIS WIZARDRY
I’ll explain for this guy.
Basically, you first find the Mach numbers for each section.
First we find the w value for M = 3 using the table B5
We find that w = 49.76 degrees.
For the top expansion, you ADD the 10degree turn to this value. Giving 59.76 degrees.
You go back to the table, and find the m value that corresponds to this angle. Which I found
to be arrooooound 3.6.
Repeat this process for all the turns. for the underside, where you’re not expanding it, you
take away the angle instead of adding. i have that M_3 = 3… can anyone tell me where i
went wrong?

Once you have your M values. You should then find the stagnation pressure, Po.
Po = P*(1+0.2*M^2) This is the formula for To, you need to use isentropic gas relations to
create the formula that relates Po to P. Gives you Po=P(1+0.2M^2)^3.5
You use the M value of the free stream before the structure.
Po = 10kPa * (1 + 0.2 * 3^2) = around 367kPa
Then you can easily find the pressures at each point using the respective mach number.
For the top surface, P = (1 + 0.2 * 3.6^2)/367
= 4.18kPa. I probably rounded differently from the guy above.
Repeat this process for all zee pressures.
what about part c?
I got P1=4.3kPa, P2=36.13kPa, P3=8.58kPa with 17.3kN lift (1)
Question 12
the shockwave is 0.577. The stagnation pressure is found to be consistent (I think) across a
a) Using the normal shock relations for a Mach number of 2, the Mach number immediately
after shockwave, therefore the inital stagnation temperature is the stagnation temperature
after the normal shockwave. T0=T(1+0.2*2^2)=396K.

b) A2/A*=A2/A1*A1/A*
A1/A* can be found from table B1 using the Mach number of 0.577 (I didn’t interpolate but
you probably should)
A2/A*=2*1.1882=2.3764
From table B1, the closest value to this was M=0.25 (again this is rough, i haven't
interpolated)
c) honestly not really sure for this section. Really could use another opinion. I got 1.18MW,
calculating a critical stagnation temp of T0*=1573.2K

d) If friction is considered, less heat would need to be added to the system, as friction will
want cause the velocities to tend to a Mach number of 1. (From memory this is due to a
Mach number of 1 corresponding to the highest entropy state of the system- don't quote me
on this though.)
12) a) To= 396k and M1=0.557 (Mo=2)
b) from B.1 with M1=0.557 A1/A*=1.22
∴ A2/A*= 2.44, which is basically M=0.25
If you expand a subsonic flow it isnt possible to end up with a Mach number of 2.5???
This leads me to conclude that M2 = 0.255 when using the A2/A* ratio of 1.22. I agree. I think
you may be right. I’ll double check working. yeah, as you can see from working below, I’ve
used 0.25, it was a typo above (2.5, 0.25…) Anywho is fixed now.
c) B.4 To1=To2=396. For M=0.25 To2/To*= 0.2605 so To*=1522K
Choked flow, To3/To*=1
q= cp(To3-To2) = 1.13MJ
d) friction will also cause flow to converge on M=1 (choked) so less heat can be added.
Orange mate above is correct, M=1 is highest entropy state so flow wants to converge
there.
a) Agree
b) How do we know if the flow will be supersonic (~2.4) or subsonic (~0.25)?? My thinking is
increasing the area of subsonic flow decrease it’s Mach number. with C I’ve seen a few
other people who got 1.13MJ but I’ll doublt check.
Doubled checked and q=1.13MJ. To1=To2= 396k, for M=0.25 To*= 1522. Since it’s choked
To3=To* so q=1004(1522-396)
1073kW/(kg/s), T0*=1464.92K, with lots of interpolation throughout
c) Assuming mach 2.4 solution, q = 156.92 kJ/kgk
d) Agree
Question 13
2009
Can someone pls post screenshots of this years exam below

Part A
1. B
2. I think the question is screwed cause can’t get near an answer, I got hf=0.09
I’m getting D using Q=0.1, maybe the translink correction Q=0.01 is for another question,
because it doesn’t work. Doing it this way gives
->v=3.183m/s, ε/D=10^-6 You can’t use this because Q is in m^3/hr, you need to convert it
first (assuming because that’s the bottom line on the moody chart, someone can correct if
they know better). (Assume it equals zero - as per Ingo.)
Re = 633452.509, so turbulent (what formula for Re was used?)
I just used Re=VD/ν, as it’s internal flow
using the Haaland’s formula -> f ≈ 0.0125
h_f = f L/D v^2/(2g) = 0.0125*1000*3.183^2/(D*2g) = 32.38m ∴D
3. A… um, you can’t even read half the question? -
U = 20m/s; x = 0.3m; y = 0.001m
use y(sqrtU/vx) = 2.1+++, which is (u/U) = 0.655++
20* 0.655++ = 13.2m/s
(A) 2
4. D
5. C <- Explanation please I got 40.09kPa (D)
Get a_1=327.69
-> Ma1=0.3998, use table B3 to find P1/P*=2.6958
-> P*=37.09kPa
->f*ΔL/D = 2.23, from table B3 (fL/D)_1 = 2.3085 thus (fL/D)_2 = 0.0785.
-> from this find P2/P* = 1.2986, thus P2=48.17kPa
So, the choke pressure is constant?
6. B
In case you get stuck on this use the equation not the tables. T0=T(1+0.2M^2)
Don’t round the mach number to 10 either, gets you A as the answer.
7. A C for the same reason as in the 2011 exam < Agree with C x3

8. Biomed shit
Part B
Question 9
a) 0.1044m -> Close enough to 10% of length, therefore safe to assume turbulence over the
whole plate.
b) 17.77mm
c) 2.22mm
d) Fd = 41.38N (for both sides of plate)
I got basically half this (and yes i multiplied by 2 for both sides of the plate) Is this the
correct way to formulate the drag interval?

Fd =2* int(0 to L=1) Cf*(½)*rho*U^2.dA


Transform dA to dx: A = b*h/2 (for one triangle half) → dA/dx =h/2→ dA = dx.h/2
Cf = 0.027/Rex^(1/7)
blabla
Fd =22.41N

I got 21.36*2 (for both sides of plate, same answer as originally) but i just used the formula
in the textbook Cd = 0.031/Re^(1/7) along with the one for Cd on the formula sheet.

e) θ = 12 +1 = 13.21
Question 10
Can someone help with 10 please?
yeerdme
The below is almost certainly wrong, help please?
a.) Yeah pretty confused with this one… Can’t think of a way to determine either p or p_0…
Ok currently thinking of working from the right hand side backwards, using the area ratio of
2.5 to find a p_0 of around 161 kPa (at Ma = 2.45ish from Table B1). Then in the reservoir,
Ma = 0, so p/p_0 = 1, so p = p_0 = 161 kPa
I think that this should be about 1600 kPa, did you divide by 0.0633
^^ I think a) is looking at a ‘just choked’ situation (which we didn’t really cover). When flow
is “just choked” at the throat, the diverging section flow is subsonic. So for a) and b) one
would use the subsonic section of B1 and for c) and d) and e) where they talk about design
conditions we’d go, well it should be supersonic. (disclaimer: i know nothing)
^^i disagree
It will exit supersonic if it chokes at the minimum point (Throat), otherwise it is subsonic at the exit

Well provided a shock doesnt form

If a shock forms then it will exit at some potato speed

How about at M = 1 p/p0 is 0.53 thus p0 = p/0.53, the external pressure is 100 therefore 100.053 is
188.7kPa???????

b.) Around 2.45ish (from above)... a=sqrt(kRT) where T comes from (P/P_0)=(T/T_0)^(k/k-1)
= 324.27. Therefore speed = 794 m/s

c.) At Ma = 1 in table B1, with p = 100 000 Pa, p_0 = 189 kPa ish. Same as back in
reservoir???
1.45MPa
I got this by finding that at M=2.45 p/p0
is 0.069, the back pressure is ideally this p, 100kPa therefore p0 is 100/0.069 which gives
1.45MPa
a is 104.1 kpa in Vince Session 2 notes :)
Ae/At=2.5because chocked M=0.25 find mach no. in subsonic part..

d.) 324.27???

e. ~9.86Mpa?
At M2=2.45 p2/p1 = 6.8 thus if p1 is 100kPa we want Pe=680KPa, from before p/p0 =
0.069 thus p0 = 680/0.069 = 9.86MPa
Question 11
a) Q = 34.84 m^3/hr agree x2 (iterate 3x to get 36.23 m^3/hr)
b)P_2 = 5.9 Pa (gauge) I agree!! Got same but negative
not really sure how you got this. Wouldn’t it make more sense to have a negative
gauge pressure as friction is reducing the energy (hence pressure) in the system? The way
i did this question (which is likely wrong) is I applied bernoulli to the system (since the flow
is assumed to be incompressible) then solved for p2 accounting for losses. This gave me a
value of approximately -0.6Kpa, Which to me seems reasonable. Would love another
opinion.
i agree here… it doesnt say, but i assume the temperature inside its lower? therefore a
lower pressure?

How did you do this!? could someone write it out…


easy, just work out f, i get f = 0.18,

Apply the energy energy equation from outside to the room:

P1/pg + (V1)^2/2g + z1 = P2/pg + (V2)^2/2g + z2 + hf, where p = rho

z1 = z2 as same elevation assumed, P1 = 0 (gauge), now P2 will come out as a gauge


pressure.
V1 = V2 as constant area

Rearranging, hopefully you get:

-f*(L/D)*((V)^2/2g) = P2/pg

P2 (gauge) = -f*(L*p/D)*((V)^2/2)

L = 15m (given), non-circular duct, so D = D_h = 4*Area/P(wetted) = 4*(40*30)/(40*2 + 30*2) =


34.286 cm = 12/35 m

p = 1.2 kg/m^3

V = Q/A = (1500/3600)/((40*30)/100^2) = 3.4722222 m/s

Check flow: Re = pVD_h/u = 79365.07937 (turbulent)

Using Haaland’s formula, with Re as above and epsilon = 0 (as smooth duct) you should
get:

f = 0.018716…

Subbing f, L, p, D = D_h and V into P2 (gauge), you get:

P2 (gauge) = -5.924 Pa, i.e. pressure in room is lower than pressure outside, makes sense
as flow from high pressure to low pressure (if anyone has another explanation, just type it
here - two explanations given above seem correct as well).
Question 12
a)1412.74 kJ/kg x1
b)No, the heat added would drive it to Ma=1? Having done part a), you should find that T2/T*
= 0.663, which corresponds to a M# of about 1.68 (Table B.4), so yes, adding the heat
amount as found in a) won’t choke the flow.
c)??? Not sure about this one either, but ended up getting M exit > 4.00 through some
dodgy working...would appreciate it if someone could explain c) and give a more
reasonable answer.
a) q= 1.415 MJ/kg (gotta cycle though T/T* to find To/To*)
b) yes it is. you only get to M=1.68. Adding heat drives it towards M=1 but you haven’t
added enough heat to do that so you’re all good. We know this cause To2 doesn’t equal
To*.
c) To2= 5775.5K
16.5/Po = 0.00316,
Po/P=316.5 = (To/T)^(k/(k-1))
To/T= 5.18= (1+(k-1)/2 * M^2)
M=4.6, T= 1115K
V=3080m/s
Flow has been accelerated which makes sense as it’s supersonic and it’s a diverging
nozzle.
Question 13
a) Image as in online problems
b) V_r1 = 4.42m/s
u_1 = 7.66m/s
w = 243.698RPM Agreex3 with all the above

c)V_r2 = 2.21 m/s


u_2 = 15.312 m/s
V_t2 = 7.064 m/s
T = 2114.96 Nm
P=53.974 kW Agreex2 with all the above

d)Q2=0.977 m^3/s
H2 = 17.23m
P2 = 164.716 kW Agreex1 will all the above
HEY CHAPS IVE ADDED 2011 DOWN THE BOTTOM IF YOU CAN CHECK IT AS WELL
^I think i love you I AGREE x2

2010
CUNT OF AN EXAM

Part A
QUESTION 1
From table 7.1 using u/U = 0.77 get
y*sqrt(U/vx) = 2.6
Solving gives x = 197.23mm I AGREE x70
therefore c

QUESTION 2
no swirl means V_t1 = 0 (alpha = 90 degrees)
smash in tan(B1) = V1/u1
V1 = Q/(2*pi*R1*b)
u1 = w*r1
solves for B1 = 60.48 degrees ------D I AGREE x7

QUESTION 3
set mg = Fd and solve for A
D = 4.957m --------C I AGREE x10
equation for A? pi*r^2 agreed

QUESTION 4
choked as pressure differential greater than 10 < where’s this from?? he says it in the
lectures or some shit?...Rob. i thought it was choked if pressure ratio > 2... ?
for expansion, use the A2/A1 to find the mach number = 2.2
then use your T/To relations, setting To = 300K
T=152K
V=Ma(KRT)^0.5
Solving gives V = 544m/s -------C I AGREE x6

QUESTION 5
Using Fig 9.23
Therefore minimum mach number is 2.5----------D I AGREE x8
do we need to know this shit considering that graph isn’t provided to us??
yeah you do, you can figure out the question using figure b1

QUESTION 6
P = rho*RT to find the rho
m_dot = rho*VA
find your mach number from your velocity, then use isentropic flow relations to find the fL/d
then
What mach number did u find? 0.323 ← This question is exactly the same as tute wk 9.
rearrange for L*
gives 10.94 which is C I AGREE x2
QUESTION 7
medical question can lick my gangrenous cunt I AGREE x >9000 +1

QUESTION 8
stupid questions I fuck’n reckon.. are you sure???? i dunno how to do it but i think ur
wrong... agree X1
Seems a bit ridiculous with all the extra detail but what comes in has to come out...
yea saw vince and he straight up said matt did it just to confuse us-Rob
Funniest shit out. Fuckin assholes! MOFO
Q1+Q2+Q3 = 0
Solve gives 30m^3/hr-----C I AGREEx8
Part B
Question 9
a)
bernoilli that mofo
gives delta z = 0.192m I Agree x (wrong but explain how)
^ Wrong as doesn’t account for frictional head loss.
=2.97m?? I also got 0.192m (what f did u use?) How did you guys get a value for f?

I found the flow was turbulent (Re = 1.173*10^6) Isn’t Re = VL/v = 1.58e9? (nope, Re=VD/v =
1.966*0.6 / (1.005*10^-6) = 1.174*10^6) then use Haalands formula to find f (assuming
smooth walled pipe(does this relate to the ‘e’ thing??) yes wat is e/D then?? zero)... found
f=0.0113... therefore h=2.96m (using frictional head loss formula) … using bernouli’s I found
that delta z = h (as V1=V2 <= V1 does not equal V2 as there is flow rate at the exit to the pipe
yes it does due to continuity... mass flow in = mass flow out (mass flow = roVA, A is
basically infinte and so V is basically 0 at start, when A decreases for pipe, V increases to
1.965)... density doesn’t change, area doesn’t change (Area goes from the area of the
storage (realy big) to the area of the pipe (small), our two points are at the top of the storage
container and the exit of the pipe. Not from the start of the submerged pipe in the storage),
therefore V1=V2....., and P1=P2), therefore delta z = 2.96m. should be 3.16 as per below
I agree with this method and the answers x11
or delta z = v_2^2/2g + h (as V_1=0 but V_2 = 1.965 ?), therefore delta z = 2.96 + 0.2 = 3.16m
((How can there be a 0 velocity at the start and 1.965 at the end??)) because (mass flow =
roVA, A is basically infinte and so V is basically 0 at start, when A decreases for pipe, V
increases to 1.965).
No. V1=V2=1.965m/s (v1 is velocity at the top of the reservoir not at the start of the pipe so
wouldn’t it be safe to assume it is negligible or = 0 ?) No, V1=V2

^^^ it says smooth walled and ignore minor losses => no f? there is f even if e/D = 0
can anyone explain more in detail how to do this what parts? or the entire thing? dont worry
got it sorted. im still confsued.. if theres no roughness how is there head loss due to
friction? Viscousity effects maybe, it uses Re so it must be something like that if you look at
haalands formula you’ll find that the 6.9/Re stays in the eqn if epsilon = 0. this gives a non
zero f value

Can someone who is saying that V1=0 explain why they think this, and also what happens
to V1A1=V2A2 - ie is there no continuity of flow?? Read purple above as well
Imagine the streamline going from the surface of the water in the tank - down through the
pipe and out the other end. The velocity of the surface of the large tank is pretty much zero
because the tank is large compared to the rate of water leaving it (thinking back to
mech2410).

A cow says....moooo, does the cow mooo for you?

LETS TRY THIS AGAIN SHALL WE (Jake):


Q = 0.555m3/s ----> V = 1.964
Re = 1.964*0.6/1.005E-6= 1.172E6
hf = fLV^2/2Dg where f = (nasty Haaland’s formula gives (with epsilon = 0) 0.01128
hf = 2.9599
ALSO
hf = delta(z) - V^2/2g V^2/2g = 1.964^2/(2*9.81) v1 = 0 but v2 is not equal to
zero as the outlet tube is not submerged in the creek!
equating hf’s
Delta z = hf + V^2/2g = 2.9899+0.193 = 3.153 m (Nick Agrees with this V1!=V2)
seem good? I AGREE x infinity
nope. V1=V2. check out wk4 in-class solutions.
shit didnt see that creek.....wait no..... it isnt touching the creek....V1 doesnt equal v2. if it
were submerged in the creek that would be the case.
what about continuity? mass flow in=mass flow out
How would there be no velocity at the start then some at the end, WHERE IS THE WATER
COMING FROM?? preach derp first point for velocity is at the top of the tank, so velocity at
state 1 is 0. So if there is continuous flow, how does this work. A1V1=A2V2, A doesnt
change (AREA DOES CHANGE !!! THE AREA OF THE TANK IS LARGER THAN THE PIPE !!!
REFER TO FACEBOOK CHAT ON THIS
^WTF is going on?!
Wouldn’t it be fair to say that if the stormwater reservoir is arbitrarily large which is
somewhat implied, that A1V1=A2V2 holds since the area of the reservoir is approximately
infinity wrt to the pipe? Therefore V1=0 and V2=Q/A? (I didn’t follow any of the other
discussions on this btw)
Can someone please go through the process for Q9(a) ?
FML
deltaZ=v^2/(2g)+hf =. v^2/(2g)*(1+f L/D)
where f = (-1.8log(6.9(kinematic viscosity of water)/(Q/A)L)^-2
Agree with above, then use Energy Equation (head form) on the formula sheet to find z
b) ~0.166m??

used bernoullis, found deltaP/(rho*g) = V^2/2g(fL/D + sumK) = h_f + h_m


sub in appropriate values, therefore f/D = 0.155 (i)
guess f = 0.03
therefore D = 0.1935m, and e/D = 0.00024
Then, reynolds = 19354.84
Sub into Haalands formula, find f... f=0.0264
sub f back into (i) and iterate until answer converges
Finally, f converges to f=0.027
sub into (i), therefore D = 0.174m (D is the width in this case) i Agree x8

is velocity constant for this? therefore Re also being constant?


how are we meant to do this iteratively if we dont have a moody diagram? like i said... guess
an inital f, subsitute into (i) to find D, find the reynolds with the new D, sub into Haalands to
find the new f... and continue this process until f converges, which is about 0.027
Question 10
a)x=0.3375m AGREE
b)d=2.63mm ← I had 2.386mm Did you use Blasius? .. AGREEx3
(blassius) Maybe not, is that what we should be doing?
turbulent gets 8.281mm? which do we use? its not like it suddenly jumps up. Its not
turbulent
Vince said you use laminar due to it still being laminar up to this point, beyond that it
begins to transition, not turbulent straight away - but does that mean we can use either
blasius or lam?......blasius is laminar exact? yep.....rob
c)tw=0.612 Pa This value corresponds to laminar Momentum Integral
I got tw = 0.278Pa (using Blasius) … i can get your answer if I use the momentum integral
approach and then multiply by 2 but not too sure about that... well remember that there are
2 sides. true shit, cheers mate
turbulent gets 1.227Pa? which do we use?

Once again i think we use blasius exact, so i get tw = 0.278Pa per side, so 0.556 Pa......Rob..
AGREEx7 --> are we usin blasius or momentum integral
d)0.01N
how did you get 0.01N?
2.02N using turbulent tw should be double this because of top and bottom
Okay, for this one i evaluated the integral form 0-0.3375 using blasius exact(laminar), then i
evaluated the integral from 0.3375-2.5 using turbulent. getting a total F = 3.59....Rob
-AGREEx2
l
anyone feel like typing out the integral? I can’t seem to get close...ended up with 2.39N

Flam = int(0.664*0.5*1.2*22.22^2*0.75*22.22^-0.5*(1.5*10^-5)^0.5*x^-0.5)dx (from 0 – 0.3375)

Fturb = int(0.027*0.5*1.2*22.22^2*0.75*22.22^-(1/7)*(1.5*10^-5)^(1/7)*x^-(1/7))dx (from 0.3375–


2.5)

Fd,total = 3.5477 N AGREE x 1


I got 3.59N, at this stage I’m inclined to say close enough

I think if there’s a question like this, explain the process then come back to it. Way too
fiddly unless you have bulk time left.

If it’s the case that you can assume turbulent flow if transition occurs at <10% of length, can
we just integrate the whole thing as turbulent? Yep but that wasn't the case here
unfortunately
Question 11
a)
expansion fans on top surface and an oblique shock on the bottom of where the airfoil is
initially hit by flow.
where the bottom part goes back flat will have expansion fans
Vince said he didnt care about the end of the airfoil as its funky and shit
I Agree x2

b)
this shit gets real
For Oblique Shock (angle = 20 degrees) at the entrance
find p2/p1 from fig B2 using M = 3
gives p2/p1 = 3.6, knowing p1 = 10kPa
Gives p2 = 36kPa
I Agree x1

For expansion fans at the bottom of the body


okay ive done some changes. Across the oblique shock i get P2/P1 = 3.6, using P1 = 10
gives P2 = 36kPa.
With M = 2 and P = 36, isentropic flow tables gives P/Po = 0.1278
solves Po = 281.69kPa (this doesnt change across an expansion fan)
Expansion Fan stuff now, using M = 2, gives w = 46.38
you plus the 20 degrees onto this thne find the mach after the expansion
M = 2.83
Isentropic flow tables for mach 2.83 flow gives p/po = 0.03572. po hasnt changes as the
expansion is isentropic, giving p = 10.062..............Robdog
M = 2(found using isentropic flow tables)But how??? mach number was found across the
shock?.....Rob
Using Pradtl-Meyer tables with M = 2 gives w = 26 degrees
but its moving back an angle of 20 degree, so its w = 46 degrees
then looking up the table again with w = 46 degrees gives M = 2.82

using M = 2 using Isentropic flow relations find p/po = 0.1278


po = 78.25kPa
then using Isentropic flow tables for M = 2.82 gives p/po = 0.036
Meaning p = 2.817
AGREE x1
For top expansion fans
Table B5 for M = 3 flow
gives w(M1) = 49.76 degrees
therefore the w(M2) = 49.76+10 degrees
Looking up these tables again using w = 59.76 gives M = 3.58

Isentropic flow tables for M = 3


yields p/po = 0.0272
p = p1 = 10kPa
therefore po = 367.65 kPa
Using Isentropic flow tables for M = 3.58 yields;
p1/po = 0.01172(Interpolating)
using the stagnation pressure i found then yields p1 = 4.308858
I Agree x4

OKAY! to find the total lift, i pretty much used the pressures i found, and used trig to find
the y component only, which would by the lift as a pressure.
First i got the differential pressure from the p_infinity
p1 = 5.69 (i think i may have done this wrong, the 10kPa acts down yea, so the pressure
is less than the atmospheric, not sure whether its 10-4.316 or the other way around)
p2 = 26 36? (i did 36-10)
p3 = 7.183
then the trig shit
p1 = 5.69 * cos(10)
p2 = 26 * cos(20)
p3 = only acts up = 7.183kPa
Vince said you dont do this cause the pressure has changed now, has nothing to do with
the initial pressures. These pressures that i calculated act normal to the surface, so just trig
that shit and summation to get your total pressure acting on the body

p1 = 4.31 * cos(10)
p2 = 36*cos(20)
p3 = 10.062 acting up

p1 acts down, both the other ones act up, so its just p2 + p3 - p1 = 39.646 acting up

Area of the wing is 1m^2


Pa = N/m^2
Therefore F_Lift = 39.646kN I agree x 1
Im not sure about this guys, please help with this.........Rob ← This would be a question I
wouldn’t do out of the 5..
Question 12

a) 396K & Man2 = 0.577 AGREE x4

Found using the initial mach number and temperature, using T/T0 (the stagnation
temperature doesnt change over the shock)

b) 0.25 (0.262)

Knowing M1=0.5774 (from normal shock) we can find values for A2/A*. Using:

A2/A*=A2/A1 x A1/A* we can find the area ratio, therefore the mach number
A2/A* = 2.0? - How is it 2.49?
I got M = 0.242 using interpolations, my A2/A* was 2.49 - I AGREEx2

how did you guys get 0.25? I thought A2/A* = 2, looked at table B1 and found M2 =
0.31(approx) ???

A2/A*=A2/A1 x A1/A* = 2*1.245 = 2.49


Then looking in B.1 @ A/A* = 2.49 → M = 0.24 approx.

c) 1.13MJ
ehhh I get like 748kJ
T02=T01, to find T03 we can use the T0/T0* values and the fact that there is a Mach number
of 1 at the exit
how was this done? As below

T02 = T01 = 396 K


@ M2 = 0.2559---> T02/T0* = 0.27047 -----> T0* = 1464.09 ← What M2 value was used
here?
Using M2=0.242, T02/T0* = 0.2464
for the flow to be choked at the exit T0* = T03
T03 = T02 + q/cp
q = 1072364.95 = 1.07 MJ/kg (Let me know what you think?) Yea
this looks good I AGREEx4

q = 1.21 MJ/kg, using values in orange font.

d) Friction would also accelerate the flow, this along with the heat would cause the flow to
choke before the exit. Therefore if friction was taken into account less heat could be added.
Question 13

Is anyone even doing this question??


I havent yet, but ill need to smash some shock tube questions :P......Rob
I dont like the look of this question....
2011
Part B
Question 9
a) 0.27m - All turb AGREEx1

b) 0.36m AGREEx1 -someone explain plz

Didn’t you just do Blasius here? Re = (130000*30/3600)/(1.5*10^(-5)) = 72 222 222


δ/x = 5/sqrt(Re)
δ = 5x/sqrt(Re)
δ = 5*30/sqrt(72222222)
δ = 0.017 m

Again we just proved it to be mostly turbulent


I bet you cant divide pi by 16.8 in your head to 5 decimal places. ← you win.

c) 1.59Pa per side AGREEx1

Again, I’m getting something different.


Tau_W = Cf * (1/2) * rho * V^2
Tau_W = (0.664/sqrt(Re)) * (1/2) * 1.2 * (130000/3600)^2
Note: Re = 72 222 222
Tau_W = 0.06113 N/m^2

It’s different because you have to use turbulent not blasius. Can you confirm the 1.59 red text
person?? Yes you get 1.59Pa per side if you use 0.027/Re^(1/7) for Cf. Cool

what’s the rule for when you use Blasius or not?

Oh!!!! Thanks! Good luck tomorrow guys, this exam looks pretty easy! I always penis up
values, and its now Multi choice

d) 390 N - theta = 66.2degrees I got 334N and 69.25 degrees? Me too

Drag Force, Fd = C_D * (1/2) * rho * V^2 * A


C_D = 2 * C_F
Fd = C_F * rho * V^2 * A
Fd = (0.664/sqrt(Re)) * 1.2 * (130000/3600)^2 * (30*3)
Fd = 11 N ← That answer looks unrealistically small. I think you have not integrated
over the length of the sign.

Fd = 167.22 N
mg = 882.9 N
theta = tan^-1 (882.9/167.2) = 79.28 deg AGREEx3
Did you guys remember to multiply Fd by 2 (because two sides)? I got exactly double (334N) Theta is
69.25
Question 10

a) 396K AGREEx2

b) M1=1.5, b1=44 AGREEx2

c) M2=0.7 AGREEx2

d) M3=0.189 AGREEx2
can somebody explain how to do this???
Using Area relations
A3/A* = A3/A2 * A2/A*
knowing M2=.7 A2/A*=1.0944 taken from table b.1
Solve for A3/A*
Then using table b.1 solve for M3=.189 (Can someone confirm this?) Yea looks good can I
just ask what A* is? <--- A* is the A that causes choking (M=1).

Refer to Lecture 8, Slide 9 - Compressible Flows. The flow accelerates over a diverging
duct. Therefore, M3 > M2 and M3=0.189 cannot be correct.

State: 2-3 (isentropic)


A3/A2 = 3
M2 = 0.7011
Need A3/A*
Therefore: A3/A* = A3/A2 * A2/A* = 3 * 1.0944
A3/A* = 3.2832
Referring to Table B.1, interpolate between Mach 2.7 and 2.8 (you know it’ll be supersonic)
and solve for M3.
M3 = 2.7316 <- you never have supersonic flow going into a combustion chamber

Agree? Yes, I agree, yes +3

Don’t agree, point 3 should be slower. <--- Don’t agree.


Point 3 is definitely slower. You’ve increased the area the flow is covering and since M2 is
sunsonic increase in area will cause the flow to slow down. I get M=0.189 same as above.

e)approximatly M4=1, very close to choking how? I get M=0.6568 (same method, different
M3=0.18)
Use Table B4, and T0/T*0. Find T*0 from T0 at the start of the combustion area.
Find the T02 from q=Cp(T02-T01) and then do T02/T*0 and use Table B4 to find the
corresponding mach number for the exit.

To1=To3=396k
For m3=0.189 To3/To*=0.1584
To*=2500k
q=2*10^6= cp(To4-To3)
To4= 2387k
To4/To*= 0.9547, corresponds to M4=0.7834
f) The expansion slowed it down a great deal. this cant be true, its compressible flow. the
expansion increases the speed (only for supersonic, we have subsonic flow at 2).if the area
stays the same the mach number would be lower so wouldn’t it be easier to choke it? The
Mach number would be greater (as M2>M3) and so less energy is required to choke. After
the shock at 1-2 the flow became subsonic. The heating section increases the flow to
eventually choke at M=1. If you don’t have the expansion, you start the combustion with a
mach number a lot closer to 1 as the expansion slows the subsonic flow. As a result, you
can’t add as much heat before choking the flow if you don’t expand it.
No it wouldn’t. Extra area dropped your mach number from 0.7 to 0.189. If it stayed at 0.7,
this reduces your To* to 435.88k. You’d choke the hell out of the flow. Increasing area of the
flow dramatically increases To*, allowing extra heat addition.
Question 11
ai) derp

ii) 498.5 this seems uber high, i got 58.87 rad/s im pretty sure its 498.86 RPM =
52.24rad/sec how ‘sure’ are we talking? pretty sure man yeah. I thought I was probably
wrong anyway

ii) P=853kW, hp = 58.24m what is the method for this question? plus numbers.... yep, I got
the same values
b) 2.84m^3/s, hp=0.46m can somebody explain how to get hp?
Question 12
Working please?
a) 13.76m i got 13.1576... not sure whos right but i double checked mine say if u agree yeah
I got 13.13 sicklad i got 13.18 - but the differences in our answers would be due to rounding
i’d say.
M2=3
From normal shock tables M3=0.4752
Interpolate in friction tables to find fL*/D=1.3765
f=0.02, D=0.2m so L*=13.765m

b) 3.94 ← 3.87 anybody else? 3.87

c) 10.2, 9.544

d) 1906 kPa - very high (Please Show working for this???) working out would be lovely
thanks!

Just work from pe, finding p* over friction areas and p2/p1 over the shock - Elaborate
please

e) The normal shock in the question would move closer to the start of the tube. Lecture: 6
on compressible flows, slide 18 (the one with the graph).

You might also like